Você está na página 1de 30

A. REPRODUCTIVE MODULE 1. A 27-year-old woman is examined by her gynecologist.

Upon rectal examination, a firm structure, directly in front of the rectum in the midline, is palpated through the anterior wall of the rectum. This structure is the: cervix of uterus (correct answer, your response) 2. As early as 8th week fetal life, the interstitial Cells of Leydig start producing: Testosterone (correct answer, your response) 3. A thick, deep-staining glycoprotein membrane that envelops the ovum in the ovarian follicle: zona pellucid (correct answer, your response) 4. M.M. a 34 year-old female residing in a remote barrio in a mountain seeks consultation accompanied by patients tall & thin parents for the first time with a chief complaint of hand injury, left with multiple bruises of the body due to fall. Upon complete physical examination, patient is noted to be of short stature for age & race, broad chest, inverted nipples, low set of ears with traces of ear abnormal discharges, webbed neck & claims with primary amenorrhea upon detailed menstrual history taking. She was subjected to left hand radiograph taking & proper wound care. Hand X-ray reveals no fracture but you are expecting for a possible incidental finding regarding the patients left hand that could add-up in diagnosing the patient also with 45X karyotyped named syndrome. 4th metacarpal, shortened (correct answer, your response) 5. The most inferior extent of the peritoneal cavity in the female is the: Rectouterine pouch (correct answer, your response) 6. In the ovary, precursors of testosterone are secreted by the: cells of the theca folliculi (correct answer, your response) 7. The male pelvis tends to differ from the female pelvis in that the male pelvis often has a: smaller subpubic angle (correct answer, your response) 8. W.G. a 26 year-old female had her 2nd imminent non-instrumental abortion because of idiopathic reason. The patient decided to seek consultation to an Ob-Gyne department, where you are currently rotating as a 3rd year ward work student. She was directly subjected to a vaginal ultrasound. Two days after, your consultant endorsed to you the typing of the result; finding & diagnosis of patient W.G. ultrasonograph, inadvertently the Ob-Gyne consultant is out for close door meeting & the computerized final printed result is due today but you cannot decipher nor read the handwriting of your consultants diagnosis. What you did is to skim & scan again the findings that say presence of pronounced indentation of the uterus fundus. What is your nearest initial impression given that ultrasonographic finding? Arcuatus (correct answer, your response)

9. Which part of the female external genitalia is the homologue of the male scrotum? labia majora (correct answer, your response) 10. The dartos fascia of the scrotum is a derivative of which of the following layers of the anterior abdominal wall? Scarpas fascia (correct answer, your response) 11. TRUE of the prostate gland: All of the above (correct answer, your response) 12. After giving birth, a patient complains of urinary stress incontinence characterized by dribbling of urine with an increase in intra-abdominal pressure. Her physician suspects injury to the pelvic floor during delivery which may have altered the position of the neck of bladder and the urethra. Which muscle was most likely damaged during the vaginal delivery? Levator ani (correct answer, your response) 13. The acrosomal vesicle transforms into an acrosome (acrosomal cap) during which phase of spermiogenesis? acrosomal phase (correct answer, your response) 14. Stereocilia are present on the luminal surface of the epithelial cells of the: ductus epididymis and ductus (vas) deferens (correct answer, your response) 15. What part of the ischioanal (ischiorectal) fossa extends deep to the sacrotuberal ligament? Posterior recess (correct answer, your response) 16. Accounts for about 10% of the volume of the male ejaculate: sperm cells (correct answer, your response) 17. Which of the following structures of the female reproductive system is the homologue of the bulb of the corpus spongiosum of the penis? Bulb of the vestibule (correct answer, your response) 18. A primary spermatocyte differentiates into a secondary spermatocyte during: meiosis I (correct answer, your response) 19. Which of the following provide dynamic support to the uterus? Pelvic diaphragm (correct answer, your response) 20. TRUE of the cervix: The functional layer of its mucosa is not shed off during menstruation. (correct answer, your response) 21. Which statement about the pelvic floor is NOT correct? It is tensed during defecation (correct answer, your response) 22. The usual medical diagnosis for XY bilateral gonadal dysgenesis or absence of functional gonads to induce puberty Swyer Syndrome (correct answer, your response) 23. The biggest cells in the seminiferous tubules: Sertoli cells (correct answer, your response)

24. When is the functional layer of the endometrium shed? start of the menstrual cycle (correct answer, your response) 25. In a CT scan of the pelvis, the uterus is located: posterior to the bladder and anterior to the rectum (correct answer, your response) 26. Which of the following parts of the prostate gland is related to the neck of the urinary bladder? Base (correct answer, your response) 27. Which of the following histologic layers is ABSENT in the wall of the seminal vesicle? submucosa (correct answer, your response) 28. The following are part of the thermoregulatory system of the testis, EXCEPT: Testicular artery (correct answer, your response) 29. To relieve perineal pain during childbirth, a pudendal nerve block is performed. Which of the following structures serve as an anatomic landmark in administering the anesthetic? Ischial spine (correct answer, your response) 30. In oogenesis, which process is arrested in metaphase and completed only by the developing oocyte if fertilization occurs? second meiosis (correct answer, your response) 31. The endometrium is at its thickest during which phase of the endometrial cycle? secretory phase (correct answer, your response) 32. Results from the merging of the ductus (vas) deferens and the duct of the seminal vesicle: ejaculatory duct (correct answer, your response) 33. Period of indifferent gonad 7th week until 18th week age of gestation (correct answer, your response) 34. The smallest of the immature gametes: spermatids (correct answer, your response) 35. A condensation of fibrous tissue in the female located at the center of the posterior border of the perineal membrane is the: Perineal body (correct answer, your response) 36. The posterior 2/3 of the scrotum is supplied by which spinal segment? S-3 (correct answer, your response) 37. The primordial germ cell that arises from the endoderm of the yolk sac is earliest recognizable stem cell of the: both (correct answer, your response) 38. The female homologue to males prepuce Hood (correct answer, your response) 39. Patients diagnosed with Testicular Feminization Syndrome have a 46 XY chromosome complement but have the external appearance of normal females, because: Insensitivity of male genital mesodermal tissue receptors to androgen

(correct answer, your response) 40. Which skeletal feature would you consider to be most characteristic of the female pelvis? Subpubic angle of 90 degrees or greater (correct answer, your response) 41. The female gamete during ovulation is a/n: secondary oocyte (correct answer, your response) 42. Which of the following cells are in the adluminal compartment of the seminiferous tubule? spermatids and secondary spermatocytes (correct answer, your response) 43. The Anlage of Cremasteric fasciae & muscles: Internal Oblique fascia (correct answer, your response) 44. During a vaginal delivery, a surgeon performed median episiotomy in which he cut too far (i.e., through the perineal body into the structure immediately posterior). Which perineal structure did he cut? External anal sphincter muscle (correct answer, your response) 45. Which of these features of the anal canal serves to indicate the point where the mucosal covering of the gastrointestinal tract ends and a skin-like covering begins? Pectinate line (correct answer, your response) 46. My beloved 33 y.o./Female cousin bare a bouncing baby girl for the 8th time singleton & sadly would like to file a case against her sister-inlaw who is both ultrasunologist & an Obstetrician that kept saying & giving false hopes of having a boy every during 3rd month of pregnancy ultrasound takings for 8 straight times. She seeks consultation in our institution for medico-legal advice. Accidentally, because you are wearing a white medical uniform, she directly asks you her first concern What is the earliest perfect timing in identifying the probable/possible sex of the fetus during gestation via ultrasound? 18 weeks age of gestation at least (correct answer, your response) 47. Which of the following vaginal fornices is the deepest? Posterior (correct answer, your response) 48. Which of the following ducts is partly extratesticular? ductuli efferentes (correct answer, your response) 49. Which structure is found only in males? Rectovesical pouch (correct answer, your response) 50. If the venous drainage of the anal canal above the pectinate line is impaired in a patient with portal hypertension, there may be an increase in blood flow downward to the systemic venous system via anastomoses with the inferior rectal vein, which is a tributary of the: Internal pudendal (correct answer, your response) 51. Which of the following lies immediately anterior to the vagina? External urethral orifice (correct answer, your response) 52. The thickest layer of the uterus is the: myometrium (correct answer, your response)

53. All but one, are the accepted reasons of most embryologists regarding the Descent of the Testis: A part of the physiologic herniation & return (correct answer, your response) 54. All but one, are signs & symptoms of patients with Reifenstein Syndrome: Anorchism (correct answer, your response) 55. Following pregnancy and delivery, a 32-year-old woman continued to have problems with urinary incontinence which developed during pregnancy. Her obstetrician counseled her to strengthen the muscle bordering the vagina and urethra, increasing its tone and exerting pressure on the urethra. This physical therapy was soon adequate to restore urinary continence. What muscle was strengthened? Puborectalis (correct answer, your response) 56. TRUE of the vagina: All of the above (correct answer, your response) 57. While you are rotating as a surgery junior intern in the out-patientdepartment, luckily you have only 2 hernia patients for the whole morning duty. The first patient is a 3 year-old, male with unilateral scrotal hernia that was diagnosed in other hospital with Scrotal Hernia, Right, with possible sigmoid content. The second patient is also a 3 year-old, female with a unilateral hernia, bulging underneath the left labia majora who sought consultation for the first time. Your senior intern asks you if you still remember the female homologue to males Processus Vaginalis. And you proudly answered: Canal of Nuck (correct answer, your response) 58. Which of the following cells are found outside the seminiferous tubules? interstitial cells of Leydig (correct answer, your response) 59. TRUE of the mammary gland: All of the above (correct answer, your response) 60. In the spermatozoon, the mitochondrial sheath forms part of the: middle piece (correct answer, your response) 61. The bulk of the lactating mammary gland is formed by: alveoli (correct answer, your response) 62. The central structure of the spermatic cord: ductus deferens (correct answer, your response) 63. Cells whose lateral surfaces form folds that partially enclose the developing male gametes: Sertoli cells (correct answer, your response) 64. Congenital Adrenal Hyperplasia can cause: Female Pseudohermaphrodism (correct answer, your response) 65. The prepuce of the clitoris is formed by the: Lateral laminae of the labia minora (correct answer, your response)

66. Incomplete cytokinesis characterizes the divisions of the developing: sperm cell (correct answer, your response) 67. Based on the zonal anatomy of the prostate gland, which of the following is most likely the site of prostatic carcinoma? peripheral (correct answer, your response) 68. TRUE of the bulbourethral gland of Cowper: They are mucus-secreting and Connective tissue septae from its capsule divides the gland into lobules. (correct answer, your response) 69. Which of the following structures is medial to the ductus deferens as it enters the anterior abdominal wall? Inferior epigastric artery (correct answer, your response) 70. The dorsal nerve of the penis is the terminal branch of: pudendal nerve (correct answer, your response) 71. The middle lobe of the prostate is separated from the posterior lobe by which of the following structures? ejaculatory ducts (correct answer, your response) 72. Which phase of the endometrial cycle is characterized by a severalfold increase in thickness of the endometrium because of mitosis of the epithelial cells and increase in amount of ground substance: proliferative phase (correct answer, your response) 73. ANALOGY = XY: _____________ = XX: fallopian tube: Appendix Testis (correct answer, your response) 74. The ovarian follicle transforms into what structure immediately after ovulation: corpus luteum (correct answer, your response) 75. Erectile tissue consists of a labyrinthine system of variously shaped vascular channels that are lined by unfenestrated endothelium. Which of the following is/are made up of erectile tissue? all of the above (correct answer, your response) 76. Which structure is NOT found within the true pelvis? Hypogastric nerve (your response) 77. Testicular duct with the thickest muscularis layer: ductus (vas) deferens (correct answer, your response) 78. At which part of the uterine tube does fertilization of the oocyte usually occur? Ampulla (correct answer, your response) 79. The ovarian vessels, nerves and lymphatics pass to and from the ovaries within which of the following structures? suspensory ligament of the ovary (correct answer, your response) 80. An anteflexed uterus describes the relative position of the body of the uterus relative to the axis of the: cervix (correct answer, your response) 81. Consists of a network of irregular anastomosing canals within the mediastinum testis: rete testes (correct answer, your response)

82. Which of the following ligaments terminate in the subcutaneous tissue of the labia majora? round ligament (correct answer, your response) 83. The perineum is bounded by all of the following skeletal elements except: spine of ischium (correct answer, your response) 84. The main constituent/s of the body of the epididymis? ductus epididymis (correct answer, your response) 85. TRUE of the oviduct or Fallopian tube: All of the above (correct answer, your response) 86. The ovarian follicles are embedded in which region of the organ? cortex (correct answer, your response) 87. While performing a hysterectomy, the resident must ligate the uterine artery. To avoid iatrogenic injury to the ureters, she must be aware that the ureter passes ___________ the artery at the level of the ______________. Under; cervix (correct answer, your response) 88. An ovarian follicle that consists of an oocyte that is surrounded by a single layer of cuboidal granulosa cells is a: primary follicle (correct answer, your response) 89. The arcus tendineus levator ani is a thickening of fascia of the: Obturator internus (correct answer, your response) 90. Which of the following cells is haploid? secondary oocyte (correct answer, your response) 91. The boundaries of the perineum include all the following except: Sacrospinal ligament (correct answer, your response) 92. G.R., an 18 year-old male reveals deficits in simple executive functions and sought consultation in your clinic with a chief complaint of gynecomastia. Patients physical examination reveals a youthful built & facial appearance, has a rounded body type , narrow shoulders, wide hips with some degree of gynecomastia, & upon examination of external genitalia, (+)female-type of pubic hair pattern with microorchidism. You ordered for karyotyping & other laboratory tests & to come back once with results. On follow-up, patients karyotype reveals 48 XXXY. What is your foremost initial impression? Klinefelters Syndrome, rare type (correct answer, your response) 93. Which of the following would be most likely to be damaged by a stab wound into the ischiorectal (ischioanal) fossa 2 cm lateral to the anal canal? Pudendal Nerve (correct answer, your response) 94. Meiosis is basically similar in oogenesis and spermatogenesis. However, compared to spermatogenesis, in oogenesis: The prophase stage during meiosis I take longer to complete. (correct answer, your response)

95. Based on the zonal anatomy of the prostate gland, which of the following is most likely the site of benign prostatic hypertrophy? Transition (correct answer, your response) B. RENAL MODULE 1. TRUE of glomerular capillaries: They are fenestrated. (correct answer, your response) 2. A possible reason for the ascent of the kidneys during permanent urinary system development & positioning: diminution of fetus body curvature (correct answer, your response) 3. The unsegmented mesoderm found in the embryos lower thoracic, lumbar, & sacral regions that will differentiate into Mesonephric & Metanephric kidney systems: nephrogenic tissue cords (correct answer, your response) 4. The normal number of segments in the kidney is: Five (correct answer, your response) 5. TRUE of the papillary ducts of Bellini: They are found only in the medulla. (correct answer, your response) 6. TRUE of the proximal convoluted tubule: It is entirely in the renal cortex. (correct answer, your response) 7. The narrowest part of the male urethra: Membranous (correct answer, your response) 8. Normally the left adrenal venous drainage is into the: Left renal vein (correct answer, your response) 9. Anlage of the urachus distal fibrous degeneration of the allantois (correct answer, your response) 10. Anlage of Loop of Henle: metanephric tissue caps (correct answer, your response) 11. TRUE of arcuate arteries of the kidney: All of the above (correct answer, your response) 12. Which of the following is NOT lined by transitional epithelium? membranous urethra (correct answer, your response) 13. The Renal Corpuscle: None of the above (correct answer, your response) 14. Relative anatomy of female urinary bladder, EXCEPT: Anterior to rectum (correct answer, your response) 15. What prevents the Horshoe Kidney to ascend & maintains its position at lower lumbar level? inferior mesenteric artery (correct answer, your response) 16. The Gerotas fascia of the kidney separates perinephric and paranephric fat and prevents spread of renal infection to all the following locations, EXCEPT: Deep pelvis (correct answer, your response) 17. Which of the following part/s of the nephron is/are lined by a simple squamous epithelium? parietal layer of Bowmans capsule (correct answer, your response)

18. Forms part of the filtration barrier in the kidney: basal lamina that is common to the endothelial cells of the glomerulus and pedicels of the podocytes (correct answer, your response) 19. Blood from an injured kidney will seep through the perirenal fat until it contacts the internal surface of the renal fascia Without perforating this fascia the blood could then continue to pass in what direction? Inferiorly toward the pelvis (correct answer, your response) 20. The ureter passes underneath the uterine artery at the level of the _____________. Internal cervical os (correct answer, your response) 21. The vasa recta that descend to, and supply, the kidney medulla arise from the: efferent arterioles (correct answer, your response) 22. Which of the following structures is NOT a part of the nephron: collecting tubule (correct answer, your response) 23. TRUE of the macula densa: All of the above (correct answer, your response) 24. What temporarily obstructs an ascending kidney on its way to its final position? arterial fork (correct answer, your response) 25. TRUE of the distal convoluted tubule: It is morphologically identical to the ascending thick limb of Henles loop (correct answer, your response) 26. Glomerulus + Renal excretory tubules = ________________ . nephron (correct answer, your response) 27. The pararenal fat in the kidney bed is an elaboration of: Extraperitoneal connective tissue (correct answer, your response) 28. Extrophy of the Cloaca is usually associated with: Omphalocele (correct answer, your response) 29. While you are undergoing a hospital rotation as a Pediatric Senior Intern, you are given a case of a 2day-old baby boy/patient, born in a non-institutionalized establishment assisted by a traditional hilot, was rushed to the emergency room of De La Salle University Medical Center with a chief complaint of urine flowing out of the babys navel. The Pediatrician-on-duty cannot attend to the patient right away due to numerous dying Pedia patients inside the E.R., so your Pedia Consultant instructed & entrusted you to refer first the patient to Surgery Department. After you have endorsed the patient, the Surgeonon-duty asks you Whats your initial impression? Urachal Fistula (correct answer, your response) 30. The internal urethral orifice is guarded behind by an elevation called ___________. Uvula vesicae (correct answer, your response) 31. Behind the kidney: All of the above (correct answer, your response)

32. it is considered as the non-functional rudimentary fetal urinary system at the beginning of 4th week of intrauterine life, although it has an internal & an external glomeruli: pronephros (correct answer, your response) 33. Which of the following is in the outer boundary of the renal lobule? interlobular artery (correct answer, your response) 34. Etiology of Duplication of Ureter, Partial: the early splitting of the ureteric bud (correct answer, your response) 35. The following structures are in the renal cortex: all of the above (correct answer, your response) 36. The posterior layer of renal fascia is known as: Fascia of Zuckerkandl (correct answer, your response) 37. Comprises the renal medulla: renal pyramids and renal columns (of Bertin) (correct answer, your response) 38. Evidence suggests, newborn patients who are diagnosed with Congenital Polycystic Kidney usually is mostly caused by malformed & malfunctioning part of the excretory unit specifically/particularly the: proximal convoluted tubule (correct answer, your response) 39. Which of the following cells are found between the macula densa and the afferent arteriole? extraglomerular mesangial cells (correct answer, your response) 40. Which of the following has branched tubuloalveolar and mucussecreting glands called urethral glands (of Littre) in its lamina propria: all of the above. (correct answer, your response) 41. the developing glomerulus is deeply indented on the Bowmans Capsule. The capsule is intimately/immediately continuous with what specific part of the nephron: proximal convoluted tubule. (correct answer, your response) 42. The following structures are in the renal medulla, EXCEPT: interlobar arteries (your response) 43. The ureter receives blood supply from all of the following arteries, EXCEPT: Celiac (correct answer, your response) 44. TRUE of short-looped (cortical) nephrons: All of the above (correct answer, your response) 45. What induces the metanephric tissue caps to differentiate into renal vesicles? distal most collecting tubules (correct answer, your response) 46. Relative anatomy of ureter, EXCEPT: Intraperitoneal (correct answer, your response) 47. The left adrenal gland may receive arterial supply from all the following arteries, EXCEPT: Celiac (correct answer, your response) 48. Which of the following histologic layers is absent in the ureter? submucosa (correct answer, your response)

49. The capillaries that forms the glomerulus drain into the: efferent arteriole (correct answer, your response) 50. The left renal vein: Is crossed anteriorly by the superior mesenteric artery (correct answer, your response) C. CARDIOVASCULAR MODULE 1. The free surfaces of which of the following structures is lined by mesothelium? epicardium and parietal pericardium (correct answer, your response) 2. TRUE of the endothelium: Its cells secretes von Willebrand factor (correct answer, your response) 3. Thickest histologic layer of the heart: myocardium (correct answer, your response) 4. Dense fibrous tissue that surrounds the AV and semilunar orifices: annuli fibrosi (correct answer, your response) 5. TRUE of the SA node: All of the above (correct answer, your response) 6. Pericytes are contractile cells that are associated with: continuous capillaries (correct answer, your response) 7. They consists of reduplicated endocardia and a core of dense connective tissue: AV and semilunar valves (correct answer, your response) 8. The histologic layer of the heart that is analogous to the tunica media of blood vessels: myocardium (correct answer, your response) 9. The tunica media is most developed in: medium arteries (correct answer, your response) 10. Which histologic layer of elastic arteries has 40-70 elastic lamellae? tunica media (correct answer, your response) 11. Type I (glomus) cells are chemoreceptors that comprise the parenchyma of: carotid bodies and aortic bodies (correct answer, your response) 12. Compared to the arteries that they accompany, veins have: more irregular lumens (correct answer, your response) 13. A blood vessel that is in between two sets of capillaries: portal vessel (correct answer, your response) 14. Which components of the lymph vascular system are interrupted along their course by lymph nodes: lymphatic vessels (correct answer, your response) 15. As a result of growth of the brain & cephalic folding of the embryo, the prechordal palte is pulled forward, while the heart & pericardial cavity become located from anterior central portion of the prechordal plate caudally to the primitive thorax proper. At what approximated embryonal age does the developing heart is positioned at the embryos neck region? 21 days old (correct answer, your response)

16. Another term for the distal 3rd of the Bulbus Cordis: truncus arteriosus (correct answer, your response) 17. The venous vessel that obliterates among the veins that drain into the Right Sinus Horn: Right Umbilical Vein (correct answer, your response) 18. Anlage of transverse pericardial sinus dorsal mesocardium (correct answer, your response) 19. Direction of shift of caudal portion of the heart tube inside the primitive pericardial cavity during the process of Cardiac Looping to the left (correct answer, your response) 20. During fetal circulation, a large portion of the bloodstream passes directly into the left atrium & a small portion, however, is prevented from doing so by the lower edge of the Septum Secundum known as: crista dividens (correct answer, your response) 21. The largest Aortic Arch that will give rise to the Carotid Arterial System: None of the above (correct answer, your response) 22. The left 6th Aortic Arch will give rise to: Left Umbilical Artery (your response) 23. The condition if there is persistent common atrium without any septations: Cor Triloculare Biventriculare (correct answer, your response) 24. What primitive venous system drains the fetal thoracic wall through intercostals venules? supracardinal veins (correct answer, your response) 25. Analogy: Fossa Ovalis is to Foramen Ovale; while, Ductus Venosus is to ___ . ligamentum venosum (correct answer, your response) 26. Fate of the remaining primitive veins of the Left Sinus Horn: Coronary Sinus (correct answer, your response) 27. The accepted process by most embryologists regarding the formation of chorda tendinae: thinning by bloodstream (correct answer, your response) 28. Anlage of hyoid arteries: 2nd Aortic Arch (correct answer, your response) 29. What specific part of the horseshoe-shaped plexus is destined to be the future the pump of the circulatory system & it is also called as the cardiogenic area of the plexus? the mid-anterior part (correct answer, your response) 30. The own vascular system of the human embryo appears in this age: late presomite embryonal age (correct answer, your response) 31. Primordium of Superior Vesicular Artery: Umbilical Arteries (correct answer, your response) 32. All but one, are considered shunts of the fetal circulatory system: fossa ovalis (correct answer, your response)

33. The most common type of Ventricular Septal Defect: Infracristal-type (correct answer, your response) 34. Cardiac Looping is approximated to be completed at what day of embryonic life: day 28 (correct answer, your response) 35. Blockage of which of the following arteries would lead to ischemia of the apex of the heart? anterior descending (correct answer, your response) 36. If the Ductus Arteriosus does not spontaneously close off soon after birth, it may have to be surgically ligate When clamping it, what important structure immediately behind it must be identified and saved? left recurrent laryngeal nerve (correct answer, your response) 37. A hand slipped behind the heart at its apex can be extended upwards until stopped by a line of pericardial reflection that forms the: oblique pericardial sinus (correct answer, your response) 38. A stethoscope placed over the right second intercostal space just lateral to the sternal angle would be best positioned to detect sounds associated with which heart valve? aortic (correct answer, your response) 39. Which chamber of the heart forms most of the diaphragmatic surface? left ventricle (correct answer, your response) 40. An elderly lady suffers a coronary occlusion and subsequently it is noted that there is a complete heart block (that is, the right and left bundles of the conduction system have been damaged.) The artery most likely involved is the: anterior interventricular (correct answer, your response) 41. During fetal life and sometimes persisting into the adult there is an opening between the right and left atria; this opening is called the: coronary sinus foramen ovale (correct answer, your response) 42. Which structure does NOT lie in the coronary sulcus? right marginal artery (correct answer, your response) 43. While attempting to suture the distal end of a coronary bypass onto the anterior descending artery, the surgeon accidentally passed the needle through the adjacent vein. Which vein was damaged? great cardiac vein (correct answer, your response) 44. The heart sound associated with the mitral valve is best heard: In the fifth left intercostal space (correct answer, your response) 45. Which posterior mediastinal structure is most closely applied to the posterior surface of the pericardial sac? esophagus (correct answer, your response) 46. In obstruction of the superior vena cava or inferior vena cava, venous blood is returned to the heart by an alternate route via the azygos vein, which becomes dilated in the process. Which of the

following structures might it compress as a result? thoracic duct (correct answer, your response) 47. You are attending an operation to remove a thymic tumor from the superior mediastinum. The surgeon asks, What important nerve lying on and partly curving posteriorly around the arch of the aorta should we be careful of as we remove this mass? left vagus (correct answer, your response) 48. Which of the following nerves would be most vulnerable to irritation when the tracheobronchial lymph nodes are enlarged due to a diseased situation? left recurrent laryngeal nerve (correct answer, your response) 49. Which vessel courses across the mediastinum in an almost horizontal fashion? left brachiocephalic vein (correct answer, your response) 50. A 85 year old male presented with edema of the left upper limb due to poor venous return. Examination revealed an aneurysm of the ascending aorta that was impinging on a large vein lying immediately anterosuperior to it, most likely the: left brachiocephalic vein (correct answer, your response) 51. An 8 year old boy is found to have a mid-line tumor of the thymus gland that is impinging posteriorly on a blood vessel. The affected vessel is most likely the: left brachiocephalic vein (correct answer, your response) 52. Most of the drainage of the thoracic body wall reaches the superior vena cava via the azygos vein. A notable exception is the left superior intercostal vein, which normally drains into the: Left subclavian vein (your response) 53. An enlarging lymph node gradually constricts the floiw of blood in the azygos venous arch. Which vessel would enlarge as a result of collateral drainage? Internal thoracic vein (correct answer, your response) 54. While viewing an exploratory surgery on a patient injured in an automobile accident, you see the surgeon elevate the esophagus off the vertebral bodies and look in the area between the azygos vein and descending aorta. What structure was she most likely looking for? Greater thoracic splanchnic nerve (your response) D. GASTROINTESTINAL MODULE (100/112) 1. Anatomical structures deranged in patients diagnosed with Bouverets Syndrome: the superior division of duodenum & Hartmanns Pouch 2. The most numerous and longest of the glands of the stomach: fundic glands 3. Peyers patches, which are MALT aggregates that contain numerous lymphoid nodules, characterize the lamina propria and submucosa of the:

ileum 4. Segment of the Digestive Tract where Langerhans cells form part of the lining epithelium: esophagus 5. TRUE statements regarding the Ileum, EXCEPT: Longer vasa recta than jejunum 6. In the major salivary glands, the main excretory ducts are formed by the union of: interlobar ducts 7. Of the structures that comprise the portal triad, which is lined by simple cuboidal epithelium? interlobular bile duct 8. Involves an enlargement of the colon, caused by bowel obstruction resulting from an aganglionic section of bowel (the normal parasympathetic enteric nerves are absent) that starts at the anus and progresses upwards. The length of bowel that is affected varies but seldom stretches for more than a foot or so. Most common first manifestation is newborns failure of meconium passage. Give the initial impression/diagnosis: Hirschprungs Disease 9. Refers to the curved skin grooves that demarcate the lateral borders of the rectus abdominis and rectus sheath: linea semilunaris 10. Refers to the transition between the posterior rectus sheath covering the superior three quarters of the rectus abdominis and the transversalis fascia covering the inferior quarter: arcuate line 11. Cells that are in the muscularis externa of the esophagus and the intestines and that act as pacesetter for gastrointestinal motility: Interstitial cells of Cajal 12. According to Molecular Regulation of the Embryology of the Digestive System, the mesoderm dictates the type of structure that will form through a HOX code which is induced by the expression of guts endoderms gene code, known as: Sonic Hedgehog 13. Hesselbachs triangle (inguinal triangle) is bounded by the following structures, EXCEPT: anterior superior iliac spine 14. G.H. a 44 year-old male, with a chief complaint of massive hematemesis with pertinent Personal & Social history of (+) 17-18 years of binge drinking (Rhum>Vodka). Past Medical history reveals, 3years prior to consultation (+) Abdominal CT-Scan & (+) Whole abdomen Ultrasound findings of a hobnail liver with confinement and admits non-complaint to attending physicians advise regarding cessation &/or moderation in alcohol intake. Patients massive hematemesis could be explained by the chronic engorgement/varicosity & recent rupture of

one of the collateral routes of Portal-Systemic Anastomosis, specifically: Esophageal veins 15. The anterior boundary of the epiploic foramen is bounded by the following structures, EXCEPT: gastroduodenal artery 16. All but one, are considered Portal-Systemic Anastomoses: Esophageal veins anastomosing to the Superior Mesenteric veins 17. Match the description on the left side with the structures on the right side. Enables the tongue to alter its shape = Intrinsic tongue musculature Protrudes the tongue = genioglossus muscle Depresses the tongue = hyoglossus muscle Stretches and tightens the soft palate = tensor veli palatine Retracts and elevates the tongue inward = palatoglossus muscle A major component of Pancreatic Juice: chymotrypsinogen 18. All but one, describe Gastroschisis in newborn: mostly on the lateral left side of the umbilicus 19. Match the description on the left side with the structures on the right side. Motor to the tongue Hypoglossal nerve Taste for the anterior 2/3 of the tongue Facial nerve Taste to teh posterior 3rd of the tongue Glossopharyngeal nerve Touch, pain and temperature sensation to the tongue Lingual nerve Touch, pain and temperature to the teeth Inferior alveolar nerve A hernia through inferior lumbar triangle: Petits Hernia 20. The following structures lie on the same horizontal plane as the transpyloric plane, EXCEPT: celiac trunk 21. The bile canaliculus is a small tube whose wall is formed by the invagination of the lateral surfaces of: adjacent hepatocytes 22. The inguinal trigone is referred to by the area bounded by the following structures, EXCEPT: inferior epigastric vessels inguinal ligament 23. According to the Functional Parts of the Liver, the liver has functionally independent right & left parts (portal lobes) that are approximately equal in size; where in the visceral surface of the liver is divided into 2 main lobes (i.e. right & left) by: Inferior Vena Cava fossa & Gallbladder fossa 24. The following statements are true regarding the Ligament of Treitz,

EXCEPT: The inferior attachment is at the level of T12 vertebral column 25. In the liver acinus, the hepatocytes that are most susceptible to damage due to anoxia (i.e., deficiency of oxygen) are those in: zone III 26. Direct inguinal hernias usually herniate through which of the following peritoneal fossa? medial inguinal 27. Segment/s of the digestive tract where the lining epithelium is simple columnar? both stomach and small intestine 28. The branch of the aorta that supplies the appendix arises from the: superior mesenteric artery 29. Most common site of colonic rupture: Cecum 30. In the liver, the space between the sinusoids and hepatic plates into which lymph flows are the: spaces of Disse 31. Indirect inguinal hernias usually herniated through which of the following peritoneal fossa? lateral inguinal 32. Which of the following structures is present in the mucosa of the large intestine? crypts of Lieberkuhn 33. Urine from a ruptured urethra will not spread inferiorly into the thigh because it is limited by the attachment of the: Scarpas fascia 34. Simple tubular or simple branched tubular glands whose ducts open into the bottom of the gastric pits: all of the above 35. Most of the cells that form the surface epithelium of the small intestine are: absorptive cells 36. A paramedian incision of the abdominal wall is preferred to the pararectus incision to avoid which of the following structures? thoracoabdominal nerves 37. To the left of the midline behind the seventh costal cartilage: Cardiac orifice 38. Major salivary gland/s that is/are mixed but mainly mucuos: sublingual 39. A positive sign would indicate that the inflamed appendix is in the pelvic area: Obturator 40. Crypts of Lieberkuhn are simple tubular glands that are present in the mucosa of the: both

41. Which artery supply the abdominal esophagus? Inferior phrenic 42. Segment/s of the small intestine where glands are present in the submucosa: duodenum 43. The pancreas does NOT have: striated ducts 44. Intestinal villi are fingerlike projections on the mucosa of the: small intestine 45. The supracolic compartment is separated from the infracolic compartment by the: transverse mesocolon 46. The liver parenchyma receives oxygenated blood from the branches of the: hepatic artery 47. TRUE of von Kupffer cells: They differentiate from monocytes. 48. A major component of Bile: bicarbonate ions 49. The first branch of the tortuous splenic artery that supply arterial blood to the superior part/aspect of proximal pancreatic body: None of these 50. Considered the as the First Segment of the Liver: None of these 51. The smooth muscle layer that forms the outermost layer of the mucosa of the GIT from the esophagus to the large intestine: muscularis mucosae 52. Which part of the duodenum has a mesentery on its proximal half? First 53. Approximately 70% of the blood (i.e. unoxygenated but nutrient-rich blood) that the liver receives is supplied by: None of these 54. Anatomy Surgery correlation regarding the functional segments of the Liver, the Right Anterior Lateral Segment is designated as: the 6th Segment 55. In the pancreas, the centroacinar cells are actually epithelial cells that line the: intercalated ducts 56. The following are the most likely points of constriction along the esophagus, EXCEPT: Ascending Aorta 57. The lymphatic drainage of the stomach is divided into four parts. Which of the four parts will most likely be affected by cancer? Second 58. The myenteric plexus of Auerbach is located in which histologic layer of the digestive tract?

muscularis externa 59. Identify the type of Rectoanal Atresia/defect: A female newborn patient with noted fistula to the lower vagina from the rectum associated with imperforated anus: intermediate type 60. The Efferent Lymphatic drainage of most Biliary ducts & accessory glands of Digestive System is through: superior mediastinal lymph node celiac lymph node 61. The inguinal triangle and the posterior wall of the inguinal canal is reinforced by the: conjoint tendon 62. At first the primordial stomach appears as a fusiform dilatation of the foregut in the 4th week of development. During the following weeks, its appearance & position change greatly as a result of different rates of growth in various regions of stomachs wall & changes in position of surrounding organs. Positional changes of the stomach are most easily explained by assuming that it rotates around a longitudinal axis, where the fusiform stomach rotates 90degrees clockwise, causing: the pylorus to be placed more inferiorly to the right the left side of the fusiform stomach to face anteriorly 63. The sphincter mechanism that governs the release of bile & pancreatic juice by the Ampulla of Vater: Oddi 64. Which part of the rectum is not covered by peritoneum? Lower 1/3 65. Plicae circulares or horizontal mucosal folds characterize the: small intestine 66. On the 10th week of development, the weight of the liver is approximately 10% of the total fetal body weight. Although this may be attributed partly to the presence of large numbers of sinusoids, another important factor is its haematopoietic function which produce red & white blood cells until this function subsides during the ____________ of intrauterine life, and only small haematopoietic islands remain at birth, hence, the weight of the liver is then only 5% of the total neonatal body weight. last 2 months of intrauterine life 67. The conjoint tendon is composed of the following muscles: external & internal oblique internal oblique & transversus abdominis 67. Segment/s of the Digestive Tract where the glands contain neuroendocrine cells: all of the above 68. Remnant of obliterated Vitelline Duct: Meckels diverticulum 69. A hernia involving a Meckel's diverticulum:

Littres Hernia 70. During intrauterine life & even adult life, this abdominal viscera is arterially supplied by the Celiac Trunk: None of these 71. In cases of appendicitis (an inflamed appendix), the shift of periumbilical pain felt during the initial stages of the disease to the right lower quadrant can be explained by: irritation of the parietal peritoneum 72. The following are tributaries to the inferior vena cava, EXCEPT: portal vein 73. The glands of von Ebner are serous glands in the tongue that are associated with: circumvallate papillae 74. Match the description on the left side with the structures on the right side. Lies at the area lateral to 2nd maxillary molar -Stensen's duct Exit of the submandibular gland -Wharton's duct Separates the parotid gland in to superficial and deep lobes -Facial nerve Exit of the sublingual gland -lingual frenulum Located along the lining of the oral cavity and buccal mucosa -minor salivary glands 75. Initially, the foregut, midgut & hindgut are in broad contact with the mesenchyme of the posterior abdominal wall, by the 5th week, however, the connecting tissue bridge has become narrow, and the caudal part of the foregut, midgut, & majority of the hindgut are suspended from the abdominal wall by the dorsal mesentery, where as, the ventral mesentery exists only in the region of: the Cap 76. Calots Triangle: None of these 77. Of the following histologic layers, which is NOT present in the wall of the gallbladder? submucosa 78. The ducts that directly drain the secretory acini of the major salivary glands and the pancreas: intercalated ducts 79. In the portal lobule, the central area is occupied by the: portal area 80. The left vagus nerve will eventually give rise to this nerve supply of the stomach: Anterior nerve of Latarjet 81. Segment/s of the Digestive Tract where there are glands in the

submucosa: esophagus 82. In the gallbladder, invaginations of the epithelium into the lamina propria that are simply diverticuli and are not glandular in nature: Rokitansky-aschoff sinuses. 83. How long is the adult small intestine (jejunum and ileum)? 20 feet 84. The stroma within its lobules is made up mainly of reticular fibers and cells in the following organ/s: liver 85. This part of the stomach is continuous with the right border of the esophagus: Lesser curvature 86. Cells of the glands of the stomach that produce hydrochloric acid: parietal cells 87. Refers to the midline vertical skin groove created by a subcutaneous fibrous band made of the decussating aponeuroses of the flat abdominal muscles extending from the xiphoid as far as the symphysis pubis: linea alba 88. Exocrine gland/s that is/are purely serous: parotid XXX submandibular XXX 89. In the major salivary glands, myoepithelial cells are present between the basal lamina and the epithelial cells in the: all of the above 90. Anlage of perineal body: Allantois & Pectinate line urorectal septum & cloacal membrane 91. During 10th week, Physiologically Herniated intestinal loops begin to return to the abdominal cavity and the first part to reenter & comes to lie on the left side is the: proximal jejunum 92. Which part of the duodenum is intraperitoneal? First 2.5cm 93. The most affected viscera in newborn patient diagnosed with Apple Peel Atresia: jejunum near the Ligament of Treitz 94. The left layers of the Falciform ligament plus the lesser omentum is equal to: the left triangular ligament 95. The free margin of the lesser omentum connecting the primordial duodenum & liver bud containing the portal triad will form as the roof of an opening that connects the Omental bursa with the rest of the peritoneal cavity, known as: Epiploic Foramen of Winslow

96. Anatomical structures involved in patients diagnosed with Mirizzi Syndrome: the Hartmanns Pouch & the common bile duct 97. Another term for vitelline cyst, where in both ends of the vitelline duct are transformed into fibrous cords, while the middle portion forms a large cyst. enterocystoma E. HEAD AND NECK MODULE
1. Match the structures in Column A with their fascial layers in Column B Diaphragm ansa cervicalis (correct) digastric muscle cutaneous branch of cervical plexus (correct) lesser occipital & great auricular nerve phrenic nerve (correct) supplies sternocleidomastoid & trapezius muscle supra-hyoid muscle (correct) supplies sternothyroid & sternohyoid muscles muscular branch of the cervical plexus (correct)

2. Match the structures in Column A with their fascial layers in Column B covers the longus capitis muscles pre-vertebral layer of fascia (correct) encloses the platysma superficial fascia (correct) enclose trapezius & sternocleidomastoid investing layer of fascia (correct) interval between pre-vertebral fascia and the pharynx retro-pharyngeal space (correct) surrounds thyroid and parathyroid pre-tracheal layer of fascia (correct)

3. Match the structures in Column A with their fascial layers in Column B platysma tributary of external jugular vein (correct) posterior auricular vein mandibular depressor (correct) lie along the external jugular vein & sternocleidomastoid infrahyoid muscle (correct) omohyoid deep cervical lymph nodes (correct)

carotid sheath lymph nodes (correct)

superficial cervical

10. The main muscle to the nose is the nasalis muscle. True (correct answer, your response) 11. The buccinator is the sucking, whistling, and blowing muscle. True (correct answer, your response)

4. Match the structures in Column A with their fascial layers in Column B superior ophthalmic vein viscerocranium (correct) lacrimal bone pyriform aperture (correct) entrance to the nasal vault tripod (correct) zygoma neurocranium (correct) temporal bone superior orbital fissure (correct) 5. Match the structures in Column A with their fascial layers in Column B ophthalmic branch of CN V vertical buttress (correct) infraorbital nerve horizontal buttress (correct) pterygomaxillary buttress V 1 (correct) frontal bar V 2 (correct) motor to masticatory muscles V 3 (correct)

6. Match the structures in Column A with their fascial layers in surgical demarcation of the superficial and deep lobes of the parotid (correct) Le Forte III oris (correct) 1st sphincter of the GIT tympani nerve (correct) lies opposite 2nd maxillary molar at buccal mucosa duct (correct) taste to anterior 2/3 of tongue dysfunction (correct)

Column B facial nerve orbicularis chorda Stensen's craniofascial

7. No lymph nodes in the scalp. True (correct answer, your response) 8. There are 9 branches of the internal carotid in the neck. False (correct answer, your response) 9. Major blood supply to the face is via the facial artery. True (correct answer, your response)

F. BLOOD AND LYMPHATICS MODULE 1. Of the following cells, which is the largest? Proerythroblast (correct answer, your response) 2. Which of the following structures seen in the lymph node consist of irregular spaces that are lined by endothelial cells that are associated with numerous macrophages? All of the above (correct answer, your response) 3. In the spleen, the artery of the white pulp is known as the: Central artery (correct answer, your response) 4. Which of the following formed elements of blood are very active phagocytes? Neutrophils (correct answer, your response) 5. Which blood cells increase dramatically in number in parasitic infestation? Eosinophil (correct answer, your response) 6. The neutrophil has the same late progenitor cell as the: Monocyte (correct answer, your response) 7. Which of the following precursor cells undergoes endomitosis? Megakaryoblast (correct answer, your response) 8. Tonsil that is located in the wall of the nasopharynx: Pharyngeal (correct answer, your response) 9. Tissue that consists of a stroma that is made up of reticular tissue and a parenchyma that consists mainly of developing blood cells of the different blood cell lineages: Myeloid tissue (correct answer, your response) 10. In the spleen, filtering of blood is the function of the macrophages that are associated with the: Sinusoids (correct answer, your response) 11. In which of the following lymphoid organs can you find activated lymphocytes? Lymph node (correct answer, your response) 12. Which of the following cells is NOT an Antigen Presenting Cell (APC)? Neutrophil (correct answer, your response) 13. Developing B cells acquire their antigen receptors while they are in the: Bone marrow (correct answer, your response) 14. The following are terminally differentiated cells, EXCEPT:

Monocyte (correct answer, your response) 15. The following proteins are present in plasma, EXCEPT Hemoglobin (correct answer, your response) 16. Of the following lymphocytes, which re-circulate most often? T cells (correct answer, your response) 17. The following cells arise directly from the hemopoietic stem cell, EXCEPT: CFU-GEMM (correct answer, your response) 18. In the thymus, epitheloid cells (epithelial reticular cells): All of the above (correct answer, your response) 19. In routinely prepared blood smears, microcytic RBCs are: Smaller than normal (correct answer, your response) 20. T-cell rich area refers to which region of the lymph node? Inner cortex (correct answer, your response) 21. Which of the following cells are involved in effecting a primary immune response? All of the above (correct answer, your response) 22. Lymphoid tissue comprises which region of the splenic pulp? White pulp (correct answer, your response) 23. Monocytes can differentiate into the following cells, EXCEPT: Langerhans cells (correct answer, your response) 24. Of the following formed elements, which are least numerous in normal blood? Lymphocytes (correct answer, your response) 25. The blood-thymus barrier that is formed by the endothelial cells and epitheloid cells is present in which part of the: Cortex (correct answer, your response) 26. Ovoid structure where the central area is occupied by activated B cells that are undergoing mitosis: Secondary lymphoid nodule (correct answer, your response) 27. Azurophilic granules are present in the following formed elements of blood, EXCEPT RBCs (correct answer, your response) 28. Of the following WBCs, which are most numerous in normal blood? Lymphocytes (correct answer, your response) 29. Of the following formed elements of blood, which has the longest lifespan? T cell (correct answer, your response) 30. The stroma of lymphoid tissue is formed by reticular fibers and reticular cells, EXCEPT in the: Thymus (correct answer, your response) 31. In adults, lymph nodules are present in the following, EXCEPT: Thymus (correct answer, your response) 32. Structures that consist of large collections of lymphoid tissue that contain aggregates of lymphoid nodules: Both (correct answer, your response)

33. The myeloblasts are the precursor cells of the: Granulocytes (correct answer, your response) 34. The following cells are nucleated, EXCEPT: Reticulocyte (correct answer, your response) 35. Of the following formed elements of blood, which is smallest in blood smears? RBC (correct answer, your response) 36. The hormone erythropoietin is: Both (correct answer, your response) 37. The percentage of blood volume that is accounted for by the RBCs is referred to as the: Hematocrit (correct answer, your response) 38. Afferent lymphatic vessels that are provided with one-way valves bring lymph to which of the following lymphoid organs? Lymph node (correct answer, your response) 39. Of the following cells, which is largest and most actively proliferating? T lymphoblast (correct answer, your response) 40. Which of the following cells have specific granules in their cytoplasm: Myelocytes (correct answer, your response) 41. The azurophilic granules present in WBCs are actually: Lysosomes (correct answer, your response) 42. Immune response that involves the production of antibodies: Humoral immune response (correct answer, your response) 43. Of the following formed elements of blood, which has the shortest lifespan? Platelet (correct answer, your response) 44. APCs present antigens to: Helper T cells (correct answer, your response) 45. Which WBC has granules that contain histamine and heparin? Basophil (correct answer, your response) 46. Memory T cells and memory B cells are responsible for the: Secondary immune response (correct answer, your response) 47. The thymic interdigitating cells, APCs that present self-antigens to the developing T cells, are located in which part of the thymus? Medulla (correct answer, your response) 48. Blood-filled sinusoids that are separated by splenic cords (of Billroth) comprise which region of the spleen? Red pulp (correct answer, your response) 49. Which of the following antibodies is/are in the plasma of people with type AB blood? Neither (correct answer, your response) 50. Of the following cells, which has the most amount of hemoglobin? Reticulocyte (correct answer, your response)

G. BONE AND CARTILAGE MODULE 1. Haversian systems characterize: compact bone (correct answer, your response) 2. In intramembranous ossification, bone replaces: mesenchyme (correct answer, your response) 3. The epiphyseal plate that allows for the growth in length of long bones is a: hyaline cartilage (correct answer, your response) 4. Their presence in the intercellular substance serves to differentiate bone from cartilage: calcium hydroxyl appetite crystals (correct answer, your response) 5. Cells that synthesize precursors of type II collagen fibers: chondroblasts (correct answer, your response) 6. Its cells are inside lacunae: both (correct answer, your response) 7. Results in poor mineralization of bones: vitamin D deficiency (correct answer, your response) 8. Internal remodeling of bone refers to the never ending cycle of breaking down and rebuilding Haversian systems. Which bone cells are involved in internal remodeling? both (correct answer, your response) 9. Cells that are still capable of mitosis: chondroblasts (correct answer, your response) 10. Capable of interstitial growth: cartilage (correct answer, your response) 11. Cells that communicate with each other via gap junctions that are formed by their processes in the canaliculi: osteocytes (correct answer, your response) 12. Special connective tissue that contains osteoprogenitor cells: all of the above (correct answer, your response) 13. Special type of dense connective tissue: both (correct answer, your response) 14. Results in poor production of bone matrix: vitamin C deficiency (correct answer, your response) 15. Cells that do NOT arise, directly or indirectly, from mesenchymal cells: osteoclasts (correct answer, your response) 16. Cartilage type where most of the extracellular fibers consist of type I collagen: fibrous (correct answer, your response) 17. In which zone of the epiphyseal plate are osteoblasts present? ossification (correct answer, your response) 18. Cells that releases its lysosomal enzymes extracellularly: osteoclasts (correct answer, your response) 19. In endochondral ossification, the periosteal elements that invade the central area of the degenerating cartilage are collectively referred to

as: periosteal bud (correct answer, your response) 20. The parathormone is the primary regulator of blood calcium level. The bone cells that have receptors for parathormone are the: osteoblasts (correct answer, your response) H. CELL MODULE 1. Microfilaments that form part of cytoskeleton of the cell are made up of: F-actin molecules 2. The polypeptides chain that comprise protein are assembled in the: Ribosomes 3. Cytoplasmic organelle that contain DNA and therefore can replicate itself: Mitochondrion 4. Which statement is NOT true regarding the cells of Embryoblast? The Hypoblast is composed of a layer of high columnar cell while the Epiblast is composed of a layer of smallcuboidal cells 5. Which of the following cytoplasmic organellesis NOT involved in the production of proteins that are to be exported by the cell? Free ribosome 6. Which of the following types of RNA molecules are produced in the nucleolus? rRNA 7. the segment of the DNA molecule within a chromosome that represent that DNA sequence for the production of a particular protein: gene 8. the intermembranous space of the nuclear envelope is continuous with the cavity of the rER 9. which of the following cytoplasmic organelles are involved in the digestion of particulate material that phagocytes engulf? Lysosome 10. Which is NOT a characteristic of syncytiotrophoblasts? Composed of an inner layer of multinucleated cells 11. Cytoplasmic inclusions that result from the lysosomal digestion of haemoglobin, the pigment responsible for the color of RBCs. Hemosiderin granules 12. Which of the following structures arise from centrioles? All of the above 13. Fibillar protein layer associated with the inner surface of the inner nuclear membrane and which stabilizes the nuvlear pores. Fibrous lamina 14. Region of the nucleolus that consist of rRNA molecules: Pars fibrosa 15. In the electron micrographs, the unit membrane is seen as tri-

laminar structur consisting of two electron denselayers that sandwich an electron lucent layer. Tails of phospholipids molecules 16. Site of synthesis of cholesterol and phospholipids: sER 17. which cytoplasmic inclusions consist of end_pruducts of lysosomal digestion? Lipofuschin pigments 18. Cytoplasmic organelles thet are not enveloped by unit membrane: Both ribosomes and centrosomes 19. Which is NOT a response of egg cell once the sperm cell has enter? Capacitation 20. Process involving transport of substances across the cell membrane that requires the presence of recepyor on the surface of the cell. Phagocytosis 21. Intermediate filaments that are only present in epithelial cells. Keratin 22. Cell organelle that is in the form of membrane-bound spherical body that contain oxidases: Peroxisomes 23. In terms of volume, what comprises 70% or more of the cytoplasmic matrix? Water 24. Which is NOT a result of Fertilzation? Formation of the 2nd Polar body of the sperm cell 25. The term human genome refer to all of the: DNA molecules that are present in a typical human cell 26. Birth defects are LEAST likely to be caused by: Environmental factors 27. Consists of parts of chromosomes that are in the process of producing RNA molecules: Euchromatin 28. Cell organelle wher microtubules, which form part of cyto skeleton, are assembled: Centrosomes 29. Ribosomal subunits are synthesized in the: Nucleolus 30. The contents of secretory granules of a cell are released to the extracellular space by: exocytosis I. CONNECTIVE TISSUE MODULE 1. Tendon consist of: -Dense regular connective tissue 2. Of the extracellular connective tissue fibers, which has the greatest

tensile strength? -Collagen Fiber 3. Extracellular connective tissue fibers that branch: -Elastic and Reticular Fibers 4. The most abundant glycosaminoglycan in connective tissue: -Hyaluronic acid 5. Fibroblast that are specializd to secrete the precursor of type III collagen are called: -reticular cells 6. Connective tissue cells that are specialized to store lipid in their cytoplasm: -Adipocytes 7. The main extracellular fiber in the stroma of the liver and lymphoid organs: -Reticular filber 8. Mesenchymal cells are the stem cells for: -AOTA 9. Extracellular connective tissue fiber that is made up of type I collagen: -collagen fiber 10. Modified loose connective tissue that forms the stroma of the liver, hemopioetic tissue and Lyphoid organs: -Reticular tissue 11. Connective tissue cells that differentiate from B cells: -Plasma Cells 12. Which of the following is formed by three (3) proclollagen molecules that twist around each other -tropocollagen 13. Procollagen, the molecular precursor of collagen, is secreted into the inter cellular spaces by: -fibroblasts 14. Connective tissue cells that are derived from progenitor cells that are in the bone marrow: -AOTA 15. Connective tissue cells that differentiate from monocytes -Histiocytes J. ENDOCRINE MODULE 1. Oxytocin and ADH are synthesized by cells of the: hypothalamus (correct answer, your response) 2. Pituicytes are supporting cells that are present in which part/s of the Pituitary gland? posterior lobe (correct answer, your response) 3. DHEA and androstenedione are produced by cells of which layer of the adrenal gland? zona reticularis (correct answer, your response)

4. Rathkes cysts, which contain eosinophilic colloidal material, are found in which part of the pituitary gland? intermediate lobe (correct answer, your response) 5. TRUE of the islets of Langerhans: Each consists of 2,000-3,000 pale-staining cells that form a compact mass. (correct answer, your respone) 6. The hypophysiotropic hormones are synthesized by secretory neurons and are released by the neurons into capillaries that are located in the: median eminence (correct answer, your response) 7. Chromophobes, which comprise majority of the epithelial cells in the pituitary gland, are mostly: corticotrophs (correct answer, your response) 8. The zona fasciculata of the adrenal gland produces: glucocorticoids (correct answer, your response) 9. TRUE of the B cells of the islets of Langerhans: They produce insulin. (correct answer, your response) 10. The hypophysiotropic hormones are delivered to the cells of the anterior lobe of the puituitary gland via the: hypophyseoportal system (correct answer, your response) 11. The wall of the thyroid follicle is formed by a simple epithelium that is comprised mostly by: follicular cells (correct answer, your response) 12. Consist/s mainly of axons of neurons whose cell bodies are in the paraventricular and supraoptic nuclei of the hypothalamus: hpothalamo-hypophyseal tract (correct answer, your response) 13. The part of the adenohypophysis that forms a sleeve around the pituitary stalk: pars tuberalis (correct answer, your response) 14. The production and secretion of T3 and T4 are the responsibilities of the: follicular cells interstitial cells (correct answer, your response) 15. Which of the following cells can be found in the adrenal medulla? chromaffin cells (phaeochromocytes) (correct answer, your response) 16. TRUE of the chief cells of the parathyroid gland: All of the above. (correct answer, your response) 17. In the pituitary gland, basophils produce: All of the above (correct answer, your response) 18. TRUE of the pineal gland: Both Modified neurons called pinealocytes comprise the vast majority of the cells of the organ and A striking feature of the organ is the presence of extracellular bodies called brain sand (correct answer, your response) 19. The following hormones are secreted by cells of the anterior lobe of the pituitary gland, EXCEPT: melanocyte stimulating hormone (correct answer, your response) 20. The thinnest histologic layer of the adrenal gland:

zona reticularis (correct answer, your response) K. EPITHELIUM MODULE 1. Exocrine glands with ducts are simple glands if: Their duct is unbranched (correct answer, your response) 2. Mesothelium and endothelium consist of: simple squamous epithelium (correct answer, your response) 3. Hemidesmosomes help attach epithelial cells to: the basal lamina (correct answer, your response) 4. Which of the following basic tissues is derived only from ectoderm? nervous tissue (correct answer, your response) 5. TRUE of exocrine glands: They deliver their secretion onto the covering or lining epithelium (correct answer, your response) 6. Type of epithelium that lines the urinary passages and bladder, which is essentially stratified cuboidal during the contracted state of the organs but stratified squamous when the organs are distended transitional epithelium (correct answer, your response) 7. Actin filaments comprise the core of: microvilli (correct answer, your response) 8. Contractile cells that are present between the secretory cells and the basal lamina of some exocrine glands myoepithelial cells (correct answer, your response) 9. Structures on the apical surfaces of cells that are attached to centrioles: both cilia & flagella (correct answer, your response) 10. Exocrine glands that release their secretion by exocytosis are: merocrine glands (correct answer, your response) 11. In some epithelial tissues, the basal surfaces of the most basal cells rest on a basement membrane, which consists of: both (correct answer, your response) 12. Cell to cell attachment among epithelial cells where the cell membranes of adjacent cells stick or even fuse with each other. zonula occludens (correct answer, your response) 13. Which of the following basic tissues contains abundant extracellular material? connective tissue (correct answer, your response) 14. Connexons, which are tiny tubes that allow for adjoining epithelial cells to exchange ions and small molecules characterize: gap junctions (correct answer, your response) 15. A gland where demilunes of Gianuzzi are present is a: mixed gland (correct answer, your response) L. MUSCULOSKELETAL MODULE 1. The connective tissue elements that envelop each muscle fascicle comprise the:

PERIMYSIUM 2. In which type/s of muscle cell are the filaments (myofilaments) arranged to form sacromere? : BOTH SKELETAL AND CARDIAC 3. Muscle cells that branch: CARDIAC MUSCLE CELLS 4. Results in poor mineralization of bones: VITAMIN D DEFIECIENCY 5. Sarcomere refers to the region of the myofibril that spans two: Z-LINES 6. Cells that communicate with each other via gap junctions that are formed by their processes in the canaliculi: OSTEOCYTES 7. Cells that do not arise, directly or indirectly, from mesenchymal cells: OSTEOCLASTS 8. Haversian systems characterize: COMPACT BONE 9. During skeletal muscle contraction, the troponin-tropomyosin complexes that cover the binding sites for myosin in the actin molecules are uncovered by: CALCIUM IONS 10. In which zone of the epiphyseal plate are osteoblast present: OSSIFICATION 11. Most of the named muscles in the body are made up of: SKELETAL MUSCLE 12. Special connective tissue that contains osteoprogenitor cells: ALL OF THE ABOVE (PERICHONDRIUM, ENDOSTEUM, PERIOSTEUM) 13. A motor unit refers to: A SOMATIC MOTOR NEURON AND THE SKELETAL FIBERS IT SUPPLIES 14. Cartilage type where most of the extracellular fibers consist of type 1 collagen: FIBROUS 15. Their presence in the intercellular substance serves to differentiate bone from cartilage: CALCIUM HYDROXYL APPETITE CRYSTALS 16. Stretch receptors that are in the endomysium and perimysium of skeletal muscles: MUSCLE SPINDLES 17. Cells that release its lysosomal enzymes extracellularly: OSTEOCLASTS 18. Capable of interstitial growth: CARTILAGE 19. The T-tubules are disposed around the Z-lines in: CARDIAC MUSCLE CELLS

20. The principal protein component of thin filaments (myofilaments): ACTIN 21. In endochondral ossification, the periosteal elements that invade the central area of the degenerating cartilage are collectively referred to as: PERIOSTEAL BUD 22. The term muscle fiber is synonymous with: MUSCLE CELL 23. Which of the following type/s of muscle cell exhibit alternating dark and light striations under the light microscope: BOTH SKELETAL AND CARDIAC 24. Cells that synthesize precursors of type II collagen fibers: CHONDROBLASTS 25. In intramembranous ossification, bone replaces: MESENCHYME 26. Special type of dense connective tissue: BOTH (BONE & CARTILAGE) 27. Internal remodeling of bone refers to the never ending cycle of breaking down and rebuilding Haversian systems. Which bone cells are involved in internal remodeling: BOTH (OSTEOBLASTS & OSTEOCLASTS) 28. Its cells are inside lacunae: BOTH (BONE & CARTILAGE) 29. The epiphyseal plate that allows for the growth in length of long bones is a: HYALINE CARTILAGE 30. Contraction of the cell results from the interaction of myosin with actin molecules: ALL OF THE ABOVE (SKELETAL MUSCLE CELLS, SMOOTH MUSCLE CELLS, CARDAC MUSCLE CELLS 31. Which type /s of muscle cells contains thick and thin filaments in its cytoplasm: ALL OF THE ABOVE (SKELETAL, SMOOTH, CARDIAC) 32. Cells that are capable of mitosis: CHONDROBLASTS 33. In skeletal muscles, the depolarization impulse from the surface of the muscle cells is instantaneously transmitted to the sarcoplasmic reticulum by the: T-TUBULES 34. Which type/s of muscle cell is multinucleated: SKELETAL 35. The parathormone is the primary regulator of blood calcium level. The bone cells that have receptors for parathormone are the: OSTEOBLASTS 36. In the sarcomeres, which structure represents the attachments of the thick filaments (myofilament) at their midpoints:

M-LINE 37. Junctional complexes that comprise the intercalated discs in cardiac muscle: ALL OF THE ABOVE (FASCIA ADHERENS, DESMOSOMES, GAP JUNCTIONS) 38. Results in poor production of bone matrix: VITAMIN C DEFICIENCY 39. The thick filaments (myofilaments) are anchored on dense bodies in: SMOOTH MUSCLE CELLS 40. In which type/s of muscle can you find motor endplates: SKELETAL M. NERVOUS TISSUE MODULE 1. Functionally, the great majority of neurons are: association neurons (correct answer, your response) 2. The process of a neuron that form part of a nerve fiber: axon (correct answer, your response) 3. Electrical synapses consist of: gap junctions (correct answer, your response) 4. CSF fills which of the following spaces? subarachnoid space (correct answer, your response) 5. Glial cells that surround the cell bodies of neurons in ganglia: satellite cells (correct answer, your response) 6. In the CNS, the cell bodies of the neurons are in the: gray matter (correct answer, your response) 7. In the PNS, the myelin that envelops large axons is formed by: Schwann cells (correct answer, your response) 8. Structurally, most neurons are: multipolar (correct answer, your response) 9. Nerve fibers are individually enveloped by endoneurium in the: PNS (correct answer, your response) 10. The microfilaments in neurons are made up of: F-actin (correct answer, your response) 11. Which glial cells are phagocytes? microglia (correct answer, your response) 12. A nerve or peripheral nerve consists of bundles or fascicles of nerve fibers. The connective tissue that surrounds each nerve fascicle is called: perineurium (correct answer, your response) 13. Glial cells that form the secretory epithelial lining of the choroid plexuses: ependymal cells (correct answer, your response) 14. Which axons are likewise enveloped by a neurilemmal sheath? both (correct answer, your response) 15. The following structure/s envelop/s large axons in the CNS:

myelin (correct answer, your response) 16. In chemical synapses, neurotransmitters are released via exocytosis into the synaptic cleft by the: presynaptic neuron (correct answer, your response) 17. Sensory neurons are pseudounipolar neurons, hence, their process that form peripheral nerve endings is morphologically a/n: axon (correct answer, your response) 18. Nissl bodies are present in the: both dendrite and perikaryon (correct answer, your response) 19. The Nissl body, the most distinctive feature of neurons are actually segments of the: rER (correct answer, your response) N. RESPIRATORY MODULE 1. The cartilaginous components of the trachea are derived from what specific embryonic origin? Somatic mesoderm X EndodermX 2. Pain felt in the anterior chest wall due to persistent coughing is due to the inflammatory irritation of the Parietal pleura 3. In the olfactory epithelium, the lfactory nerves are formed by the Axons of the olfactory cells 4. The intercostal neurovascular structures may be found closely related to which part of the ribs? Subcostal groove 5. What is the most common type of tracheoesphageal fistula? Small h type/ esophageal lower segment forms a tracheal fistula 6. The intercostal nerve give off a lateral cutaneous branch approx. at the Mid axillary line 7. Each of the bronchopulmonary segments are supplied independentyly by thichof the following? Both A and B 8. Final Drainage of the fontal, maxillary and anterior ethmoid sinuses Middle meatus 9. Part of the osteomeatal unit Hiatus semilunaris, uncinate process 10. Drains the maxillary sinus Facial Vein 11. Possible pathway of infection to the cavernous sinus Facial vein 12. Removed during sinus surgery to improve drainage of the sinuses Hiatus semilunaris, uncinate process 13. The limb deformity in VACTERL association mostly affects which type of bone

The radius 14. When inserting a hypodermic needle into the thoracic wall, it is best to insert it near the superior border of the rib because We want to avoid the intercostal neurovascular bundle found in the subcostal groove 15. The part of the ribs most commonly prone to fracture Portion anterior to the angle 16. Secretes calcitonin Thyroid gland 17. Clinically significant for establishing an emergency airway Cricothyroid ligament 18. Only intrinsic muscle of the larynx not supplied by the recurrent nerve Cricothyroid muscle 19. Maintain proper levels of extracellular fluid concentration of calcium Parathyroid gland 20. Adductor muscle Cricothyroid muscle 21. Aspiration of foreign body is most likely to affct the right lung because The right bronchihas a larger diameter 22. A young woman came into the emergency department with a stab wound th 5 intercostal space 23. The wall of which of the following segments of the bronchial tree has cartilage Tertiary bronchus 24. The broadest, widest and nearly horizontally angulated rib is the st 1 rib 25. When respiratory diverticulum expands in a caudal direction, it becomes separated. . . Caspase 26. Which of the following statements is TRU regarding the developmental characteristics of the st pharyngeal arch? Muscles of facial expression were derived from this arch X The stylopharyngeus muscle was derived from this arch X It is innvervatedby cranial nerve V 27. When respiration begins at birth, most of the lung fluid is rapidly resorbed by the blood and lymph capillaries . . . Vaginal contraction 28. What specific layer of the embryonic mesoderm that will differentiate into as the lungs parietal pleura? Somatic mesoderm 29. After the recanalization and revacuolization of temporarirly occluded laryngeal orifice secondary to rapid laryngeal apithelial proliferation. . .

The true and false vocal cords 30. The most abundant of the cell types that comprise respiratory epithelium Ciliated columnar cell 31. The transverse process of the vertebra articulates with which part of the ribs Tubercle 32. Precursor cells of pulmonary alveolar macrophages Monocytes 33. Epithelial cell that forms part of the blood air barrier in the lung Type 1 alveolar cell 34. A young man was thrown off his motorcycle in a vehicular accident ... A strip of skin at approximately at the level of the sternal angle 35. The capillaries in the interalveolar septa whose endothelium form part of the blood-air barrier in the lung are terminal branches of the Pulmonary artery 36. Lower portion and greater horn of the hyoid bone rd 3 pharyngeal arch 37. Orbicularis oris muscle th 4 pharyngeal arch X st 1 pharyngeal arch X 38. Ultimobranchial body th 4 pharyngeal arch X rd 3 pharyngeal arch X 39. Malleus and incus st 1 pharyngeal arch 40. Constrictors of the pharynx th 4 pharyngeal arch 41. Anterior 2/3 of the tongue th 4 pharyngeal arch X nd 2 pharyngeal arch X 42. Blends with the investing, pre tracheal and pre vertebral layer of deep cervical fascia Carotid sheath 43. Pre tracheal layer of deep cervical fascia Covers the thyroid gland 44. Formed by the anterior rami of the first 4 cevical nerves Cervical plexus 45. Greater auricular nerve Cervical plexus 46. Encircles the neck and covers the sternocleidomastoid muscle Investing layer of deep cervical fascia 47. On what specific period of lung maturation where the terminal bronchioles and the respiratory bronchioles are already formed . . . Alveolar period

48. The cartilates that form part of the wall of the trachea and the bronchi are Hyaline 49. Which of the following ribs have a prominent tubercle for the attachment of the serratus anterior muscle? nd 2 rib 50. The lingual is part of the Superior lobe of left lung 51. It is estimated by almost all embryologists that approximately . . . 1/6 52. The segment of the bronchial tree that supplies. . . bronchiole 53. Superior parathyroid gland th 4 pharyngeal pouch 54. Inferior parathyroid gland rd 3 pharyngeal pouch 55. Tympanic membrane st 1 pharyngeal pouch 56. Cell that secretes pulmonary surfactant Type 2 alveolar cell 57. Secretomotorineration is via the sphenopalatine ganglion Maxillary sinus 58. Intimately related to the optic nerve and the internal carotid artery Sphenoid sinus 59. Opens at the antero-superior portion of the osteomeatal unit Frontal Sinus 60. Has a paper thin wall easily penetrated. . . Ethmoidsinus 61. Begins development at 3 mos AOG Frontal sinus X Ethmoid sinus X Sphenoid Sinus X 62. Alveolar sacs and alveoli arise from Respiratory bronchioles and alveolar ducts 63. On what approximated AOG where the embryos ventral walls foregut will produce . . . th 25 day 64. The most common complication that could affect the pregnancy . . . Polyhydramnios 65. Which among the following statements regarding the skeletal and cartilaginous derivatives of . . . rd th The laryngeal cartilages were derived from the 3 and 4 pharyngeal arches 66. When the st pharyngeal cleft and tubotympanic recess further interact and develop, it becomes what structure External auditory canal

67. Separates the ethmoid sinuses from the anterior cranial fossa Fovea ethmoidalis 68. Drainage of the sphenoid sinus Sphenoethmoidal recess 69. Not opened routinely during endoscopic sinus surgery Spheno-ehtmoidal recess 70. Used to prognosticate maxillary sinus cancer Ohngrens line 71. Drainage of the frontal sinus Frontal recess 72. On physical examination, an abnormal breath sound was noted on the left posterior chest wall, approximately 2-3cm lateral to the paravertebral line . . . Superior segment of the left lower lobe 73. The lingula is part of the Superior lobe of left lung 74. Which of the following laryngeal cartilages is elastic epiglottic 75. Devoid of lymphatics True vocal cords 76. Voice articulator Lips 77. Only intrinsic muscle with bilateral innervation Interarytenoid muscle 78. Results from injury of the superior laryngeal nerve Bowing of the vocal cords 79. Adductor muscle Interarytenoid muscle 80. Type of cell that participates mainly in blood air barrier during alveolar period Pneumoctye type 1 81. The following is true of children below 2 years of age The AP diameter and transverse iamter of their thorax is equal 82. TRUE of the apex of the pleura B AND C ONLY 83. Which of the following statements is NOT true regarding the development of the palate? the upper jaw component of the intermaxillary segment give rise to the philtrum of the upper lip 84. at the mid axullary line, the lowermost part of the costal pleura reaches the level of the th 10 rib 85. The internal thoracic artery is best ligated below the first 2 interspaces because The sternocostalis (transversusthoracis) muscle lie behind the internal thoracic artery at the lower interspaces

86. Which of the following statements is true regarding development of the facial bones It developed from the viscerocranium 87. Of the following organs, which contains the greatest amount of malt in its mucosa Pharynx 88. Bronchialsubmucosal glands are mixed glands that are in the submucosa of the Trachea 89. The type of epithelium that lines the vocal cords Pseudostratified columnar X 90. Gestational embryonic age-range for pseudoglandular period 5 weeks 17 weeks old 91. The E in VACTERL association stands for None of these 92. Clara cells form part of the epithelium of which segment of the bronchial tree? Bronchile and terminal bronchiole 93. The muscles/ cartel;ages of the fourth pharyngeal arch are innervated by Superior laryngeal nerve 94. Which of the following statements is not true regarding the development of the face The lateral nasal prominence formts the philtrum of the upper lip 95. A stab wound 2-3cm medial to the scapular line at the level of the th 11 thoracic vertebra may injure the following structures Pleural cavity X Kidneys X Lung parenchyma X 96. The sternal angle is in the same horizontal plane as the following structures except nd Head of the 2 rib O. SKIN MODULE 1. Simple branched alveolar gland Sebaceous gland 2. Which cells in the epidermis contain the most amount of melanin Keratinocytes 3. Which cells in the epidermis arise Langerhans cells 4. Apocrine sweat glands All of the above 5. The keratin filaments that fill Intermediate filaments 6. The epithelial cells that form hair are those that comprise Hair matrix

7. A merocrine gland Sweat gland 8. Which layer of the skin is derived from ectoderm epidermis 9. The cells of which layer are mainly responsible for renewal of the Epidermis Stratum basale 10. Layer of the dermis that consists of dense irregular connective tissue Reticular layer 11. Which of the following structures are often considered appendages of hair Both sebaceous glands and arrectorpili muscle 12. Which of the following appendages are present in thick skin Sweat gland 13. The pithelial cells that form nail are those that comprise the Nail matrix 14. It consists essentially of the epidermis of the skin that was pulled downwards External root sheath 15. Vaterpacinian corpuscles and meissners Sensory nerve endings 16. Which of the following histologic layers is NOT PRESENT Stratum lucidum P. LOWER LIMB 1 AND 2 1. The femoral ring is bounded laterally by the _________. femoral vein (correct answer, your response) 2. The team doctor tells a football player that he has a pulled hamstring muscle. This results from a tearing of the origin of a hamstring muscle from the: ischial tuberosity (correct answer, your response) 3. Following a penetrating injury to the left femoral triangle, a patient related that walking was virtually impossible because at every step the left knee collapsed into flexion. This history suggests paralysis of which muscle? Adductor magnus (your response) 4. Childhood immunizations are sometimes given via intramuscular injections into the quadriceps muscles of the anterior thigh. At the midthigh level, a needle passing into the space deep to the sartorius muscle might pierce the femoral vessels as they lie in the: Adductor canal (correct answer, your response) 5. The femoral triangle is bounded laterally by the ________. Sartorius (correct answer, your response) 6. In order to avoid injury to the sciatic nerve, intramuscular injections should be given in which quadrant of the buttock?

upper lateral (correct answer, your response) 7. Inability to extend the knee and loss of cutaneous sensation over the anterior surface of the thigh would indicate a lesion or compression of the: femoral nerve (correct answer, your response) 8. The femoral artery enters the popliteal fossa (becoming the popliteal artery) by passing through the: adductor hiatus (correct answer, your response) 9. A fracture of the ischial tuberosity might be expected to most directly affect the muscles that produce which lower limb movement? flexion at the knee (correct answer, your response) 10. The femoral canal contains the: inguinal lymph node of Cloquet (correct answer, your response) 11. Which of the following anterior thigh muscles is also known as the kicking muscle? rectus femoris (correct answer, your response) 12. The largest branch of the lumbar plexus. Femoral (correct answer, your response) 13. The longest muscle in the body. Sartorius (correct answer, your response) 14. A saphenous vein cut-down is a surgical procedure that involves cutting through the skin to locate the great saphenous vein in order to insert a catheter or cannula. To find this vein, the skin would be incised: anterior to the medial malleolus (correct answer, your response) 15. Innervates the largest muscle in the body. inferior gluteal (correct answer, your response) 16. Weakness in climbing stairs or jumping would indicate a lesion of which nerve? Superior gluteal (your response) 17. It is the longest and heaviest bone in the body. Femur (correct answer, your response) 18. It is the longest vein in the body. great saphenous (correct answer, your response) 19. As a patient with paralyzed gluteus medius and minimus muscles on the left side attempts to stand on the left limb only, the right side of the pelvis typically: drops (correct answer, your response) 20. When, in approximately 12%, the common fibule nerve passes through the piriformis muscle, the nerve may be compressed. This would affect part of which muscle? Biceps femoris (correct answer, your response) 21. An obturator hernia that compresses the obturator nerve in the obturator canal may affect the function of all of the following muscles EXCEPT: Obturator internus (correct answer, your response)

22. Which statement is TRUE? The femoral vein lies medial to the femoral artery (correct answer, your response) 23. It is the longest sensory nerve. Saphenous (correct answer, your response) 24. It is also the conjoint aponeurosis of the gluteas maximus and tensor fascia lata. iliotibial tract (correct answer, your response) 25. What muscle passes through the lesser sciatic foramen? obturator internus (correct answer, your response) 26. The largest muscle in the body. gluteus maximus (correct answer, your response) 27. It is the strongest ligament in the body. Iliofemoral (correct answer, your response) 28. Injury to the _________ nerve results in a characteristic gluteal gait. superior gluteal (correct answer, your response) 29. The muscular floor of the femoral triangle is formed by _________ medially and _________ laterally. Pectineus, iliopsoas (correct answer, your response) 30. Following surgical opening of the adductor canal, a patient experienced a loss of cutaneous sensation of the medial side of the leg. Which nerve was cut? Saphenous (correct answer, your response) Q. BACK 1. Which of the following statement is NOT true regarding the ligaments and joints of the back ~ The Zygapophysial joint is nan example of a secondary cartilaginous 2. Which among the following statements is TRUE regarding the flexion of the trunk? ~ There is bilateral action of the rectus abdominis 3. Which among the following statements is TRUE regarding the Lateral bending of the trunk? ~ There is bilateral action of the longisimus thoracis 4. Which among the following is NOT part of Transverse Spinal group of muscle? ~ Interspinalis 5. Which among the following components of the Erector spinae group does not have a proper lumbar attachment? ~ spinalis R.&S. UPPER LIMB 1 AND 2 1. Innervates the largest muscle in the body. inferior gluteal (correct answer, your response) 2. The femoral ring is bounded laterally by the _________. femoral vein (correct answer, your response)

3. It is the strongest ligament in the body. Iliofemoral (correct answer, your response) 4. The femoral artery enters the popliteal fossa (becoming the popliteal artery) by passing through the: adductor hiatus (correct answer, your response) 5. When, in approximately 12%, the common fibule nerve passes through the piriformis muscle, the nerve may be compressed. This would affect part of which muscle? Biceps femoris (correct answer, your response) 6. The largest muscle in the body. gluteus maximus (correct answer, your response) 7. The longest muscle in the body. Sartorius (correct answer, your response) 8. It is the longest vein in the body. great saphenous (correct answer, your response) 9. It is the longest sensory nerve. Saphenous (correct answer, your response) 10. Inability to extend the knee and loss of cutaneous sensation over the anterior surface of the thigh would indicate a lesion or compression of the: femoral nerve (correct answer, your response) 11. It is also the conjoint aponeurosis of the gluteas maximus and tensor fascia lata. iliotibial tract (correct answer, your response) 12. The femoral canal contains the: inguinal lymph node of Cloquet (correct answer, your response) 13. The femoral triangle is bounded laterally by the ________. Sartorius (correct answer, your response) 14. Weakness in climbing stairs or jumping would indicate a lesion of which nerve? Inferior gluteal (correct answer, your response) 15. The largest branch of the lumbar plexus. Femoral (correct answer, your response) 16. Following surgical opening of the adductor canal, a patient experienced a loss of cutaneous sensation of the medial side of the leg. Which nerve was cut? Saphenous (correct answer, your response) 17. The muscular floor of the femoral triangle is formed by _________ medially and _________ laterally. Pectineus, iliopsoas (correct answer, your response) 18. It is the longest and heaviest bone in the body. Femur (correct answer, your response) 19. A saphenous vein cut-down is a surgical procedure that involves cutting through the skin to locate the great saphenous vein in order to insert a catheter or cannula. To find this vein, the skin would be incised:

anterior to the medial malleolus (correct answer, your response) 20. Injury to the _________ nerve results in a characteristic gluteal gait. superior gluteal (correct answer, your response) 21. Childhood immunizations are sometimes given via intramuscular injections into the quadriceps muscles of the anterior thigh. At the midthigh level, a needle passing into the space deep to the sartorius muscle might pierce the femoral vessels as they lie in the: Adductor canal (correct answer, your response) 22. What muscle passes through the lesser sciatic foramen? obturator internus (correct answer, your response) 23. An obturator hernia that compresses the obturator nerve in the obturator canal may affect the function of all of the following muscles EXCEPT: Obturator internus (correct answer, your response) 24. As a patient with paralyzed gluteus medius and minimus muscles on the left side attempts to stand on the left limb only, the right side of the pelvis typically: drops (correct answer, your response) 25. In order to avoid injury to the sciatic nerve, intramuscular injections should be given in which quadrant of the buttock? upper lateral (correct answer, your response) 26. Which statement is TRUE? The femoral vein lies medial to the femoral artery (correct answer, your response) 27. A fracture of the ischial tuberosity might be expected to most directly affect the muscles that produce which lower limb movement? flexion at the knee (correct answer, your response) 28. The team doctor tells a football player that he has a pulled hamstring muscle. This results from a tearing of the origin of a hamstring muscle from the: ischial tuberosity (correct answer, your response) 29. Which of the following anterior thigh muscles is also known as the kicking muscle? rectus femoris (correct answer, your response) 30. Following a penetrating injury to the left femoral triangle, a patient related that walking was virtually impossible because at every step the left knee collapsed into flexion. This history suggests paralysis of which muscle? Quadriceps femoris (correct answer, your response) T. UPPER LIMB 3 1. Also known as the intermediate layer of the Flexor-Supinator Compartment: the 2nd layer (correct answer, your response) 2. The bursa located between olecranon & Triceps brachii tendon: Subtendinous Olecranon Bursa (correct answer, your response)

3. Among the bands that consists the ulnar collateral ligament, what ligamentous band is morphologically slender which deepens the trochlear notch? Oblique Band (correct answer, your response) 4. The medial, triangular ligament extending from the medial epicondyle of the humerus to the coronoid process & olecranon of the ulna Ulnar Collateral Ligament (correct answer, your response) 5. The triceps brachii is the chief extensor muscle of the elbow joint & is assisted by a small triangular unimportant muscle on the posterolateral aspect of the elbow, called: Anconeus (correct answer, your response) 6. Regarding the collateral anastomosing arterial network of the elbow joint, how many branch/es is/are supplied by the radial artery? one (correct answer, your response) 7. A band of fibrous thickening from the flexor retinaculum bridges a groove to form a small canal between pisiform & the hook of the hamate where the ulnar nerve passes through. Clinically, what do you call this canal? Canal of Guyon (correct answer, your response) 8. The elbow joint is under what type of synovial joint? hinge type of joint (correct answer, your response) 9. All but one, are the chief flexors of the elbow joint: Anconeus (correct answer, your response) 10. All flexor muscles of the forearm are supplied by the median or by the ulnar nerves, EXCEPT: Brachioradialis (correct answer, your response) 11. Generally, pronation motion is innervated by what nerve? Median nerve (correct answer, your response) 12. Pronation is primarily produced by the Pronator Quadratus muscle & secondarily by Pronator Teres muscle, with some assistance from what muscle especially in midprone position? Brachioradialis (correct answer, your response) 13. Among the posterior extensor muscles of the forearm, what muscle originates from the lateral supracondylar ridge which is directly supplied by the superficial branch of the Radial nerve? Extensor Carpi Radialis Longus (correct answer, your response) 14. What muscle of the forearm is incapable of acting alone at the wrist joint because its muscle belly or tendon does not cross between radius & ulna or to the interosseous area? Brachioradialis (correct answer, your response) 15. What structure is considered as the roof of the Cubital fossa? Deep fascia (correct answer, your response) U. UPPER LIMB 4 1. Which of the following compartments contain the Lumbrical muscles?

Central compartment (correct answer, your response) 2. Which of the following bones articulate with either the inferior end of the Radius and / or the articular disc of the wrist joint? both Triquetrum and Scaphoid (correct answer, your response) 3. Which among the following muscles is NOT innervated by the Ulnar nerve? Abductor pollicis brevis (correct answer, your response) 4. Which of the following structures form the knuckles of the Hand? both Bases of the Proximal Phalanges and Heads of the Metacarpals (correct answer, your response) 5. Which does NOT bound the Anatomical Snuff box? both Tendon of the Adductor pollicis longus and Tendon of the Flexor pollicis brevis (correct answer, your response) 6. Which of the following statements is NOT true regarding the blood supply of the hand? The Deep palmar arch is formed mainly by the Ulnar artery (correct answer, your response) 7. Which of the following compartments is bounded by the Palmar aponeurois superiorly, the Lateral fascial sheath laterally, the Medial fascial sheath medially and the Midpalmar space inferiorly? Central compartment (correct answer, your response) 8. Which of the following statements is NOT true regarding the nerves of the hand? The Ulnar nerve supplies the cutaneous innervation of the lateral part of the 4th digit (correct answer, your response) 9. Which of the following statements is NOT true regarding the compartments of the Hand? The Palmar aponeurosis overlies the extensor tendons. (correct answer, your response) 10. Injury to the Ulnar nerve would cause:: Claw hand (correct answer, your response) V. LOWER LIMB 3 AND 4 1. All but one, are structures deranged mostly in patients diagnosed with Odonoghue Injury: Patellar Fat Pad (correct answer, your response) 2. All but one, are contents of Popliteal Fossa: Great Saphenous Vein (correct answer, your response) 3. The lateral calf cutaneous nerve plus medial calf cutaneous nerve is equal to Sural nerve (correct answer, your response) 4. The following ankle ligaments are lateral ligaments, EXCEPT: tibionavicular (correct answer, your response) 5. A 20y.o./Female patient was diagnosed with a Mild Anterior Leg Compartment Syndrome due to chronic & repetitive microtraumatic injury of the Tibialis Anterior muscle with concomitant small tears in

the periosteum covering of the tibial body incurring edema & pain of the distal 2/3rd of the pretibial area: Colloquial term for such diagnosis is: Shin Splints (correct answer, your response) 6. Clergymans Knee; inflammation of a bursa due to excessive friction of what specific bursa of the knee secondary to repetitive genuflecting & chronic kneeling while praying? Subcutaneous Bursa (correct answer, your response) 7. Among the extracapsular ligaments of the Knee joints fibrous capsule, if the patellar ligament is a continuation of Quadriceps Femoris tendon anteriorly: the oblique ligament is an extension of which muscles tendon posteriorly? Semimembranosus (correct answer, your response) 8. Compression or entrapment of what specific nerve during skiing, running or among soccer players secondary to tight-fitting ski boots or running/spike shoes, respectively, can cause a condition with symptoms of pain in the dorsum of the foot, radiating to the web space between the 1st & 2nd digits called the Ski Boot Syndrome? Deep Fibular Nerve (correct answer, your response) 9. The keystone of the lateral longitudinal arch is the: cuboid (correct answer, your response) 10. The roof of the Popliteal fossa: Skin with Crural fascia (correct answer, your response) 11. The medial, large terminal branch of Sciatic Nerve; it is also the most superficial of the three (3) main central components of Popliteal Fossa: Tibial Nerve (correct answer, your response) 12. Arterial supply of Posterior & Lateral compartments of leg which also supplies the circumflex fibular branch which joins anastomoses around knee; & of nutrient artery passing to the tibia: Posterior Tibial Artery (correct answer, your response) 13. All but one, are tendons inserted distally on the anserine bursa hence, called pes anserinus: Semimembranosus (correct answer, your response) 14. Which of the following ligaments support the head of the talus? plantar calcaneonavicular (correct answer, your response) 15. The keystone of the medial longitudinal arch is the: head of the talus (correct answer, your response) 16. The deepest structure in the Popliteal fossa which runs close to the articular capsule of the knee joint: Popliteal Artery (correct answer, your response) 17. The most stable position of the ankle joint is when the foot is: in dorsiflexion (correct answer, your response) 18. Gerdys tubercle is the distal insertion for what structure? Iliotibial Tract (correct answer, your response) 19. All but one, are considered contributors to Genicular Anastomosis:

Posterior Tibial Recurrent artery (correct answer, your response) 20. The ankle joint is a: hinge joint (correct answer, your response) 21. Among the intracapsular ligaments of the Knee joints fibrous capsule, it is the C-shaped, weaker among menisci: medial (correct answer, your response) 22. Which of the following joints is the usual site of amputation of the foot? transverse tarsal (correct answer, your response) 23. What nerve innervates the skin of the distal 3rd of anterior surface of leg & dorsum of foot? Superficial Fibular nerve (correct answer, your response) 24. Where can you usually find the sesamoid bone fabella that is visible in lateral radiographs of the knee in 3 to 5% of people? Close to proximal attachment of: Lateral Head of Gastrocnemius (correct answer, your response) 25. The Biceps Femoris tendon is split into two parts distally because of what extracapsular ligament: Fibular Collateral Ligament (correct answer, your response) 26. What nerve innervates the skin of first (1st) interdigital cleft? Deep Fibular nerve (correct answer, your response) 27. Which of the following joints of the foot allow inversion and eversion? both subtalar and transverse tarsal (correct answer, your response) 28. The commonly affected ankle ligament in ankle sprains in cases of inversion injuries of the foot is the: anterior talofibular (correct answer, your response) 29. The usual muscle deranged or injured in patients diagnosed with tennis leg: Medial Head of Gastrocnemius (correct answer, your response) 30. Pulse that can be felt posterior to the medial malleolus is from this artery: posterior tibial (correct answer, your response) W. NEUROANATOMY MODULE PART 1 1. Part of the eminences of the corpora quadrigemina and also part of the tectal region of the mesencephalon which holds/carries the principal midbrain nucleus for the auditory pathway & receives input from several more brainstem nuclei in the sense of hearing pathway: Posterior Colliculi (correct answer, your response) 2. A pure motor functioning rhombomere: CN XI (correct answer, your response) 3. A pure sensory functioning rhombomere: None (correct answer, your response) 4. That portion of the pia mater that covers the ependymal roof or, in the case of lateral ventricle, medial wall of a cerebral ventricle. This is

also the double fold of pia mater, enclosing the subarachnoid trabeculae, between the fornix above & the epithelial roof of the 3rd ventricle & the thalami below; at each lateral margin is a vascular fringe projecting into the choroidal fissure of the lateral ventricle; on its undersurface are several small vascular projections filling the folds of the ependymal roof of the 3rd ventricle: tela choroidea (correct answer, your response) 5. The presence of an additional cranial nerve (the nervus terminalis or cranial nerve zero) is well documented in many non-human vertebrate species & adult human brain. This cranial nerve was identified bilaterally as a microscopic plexus of unmyelinated peripheral nerve fascicles in the subarachnoid space covering the gyrus rectus of the orbital surface of the frontal surface. The plexus appeared in the region of the cribriform plate of the ethmoid and coursed posteriorly to the vicinity of the olfactory trigone, medial olfactory gyrus, and lamina terminalis. Question: What is the principal contributory function of Cranial Nerve Zero? smell (correct answer, your response) 6. A non-mixed functioning cranial nerve guided by rhombomere: Abducens nerve (correct answer, your response) 7. Cerebrospinal Fluid Analysis: Appearance = clear Polymorphonuclear cell = slightly increased Lymphocyte = markedly increased Protein = markedly increased Glucose = normal Viral (correct answer, your response) 8. The following general statements concerning the medulla oblongata are correct, EXCEPT: medulla (correct answer, your response) 9. The following statements concerning the anterior surface of the medulla oblongata is/are correct, EXCEPT: On each side of the midline, there is an ovoid swelling called the olive, which contains the Corticospinal fibers (correct answer, your response) 10. This occurs when there is damage to the brain caused by stroke or injury, and there may be an actual shrinkage of brain substance (brain atrophy) with normal cranial vault. Although there is more CSF than usual, the CSF pressure itself is normal in this condition. Whats your initial impression? Hydrocephalus ex vacuo (correct answer, your response) 11. Lissencephaly: smooth brain (correct answer, your response) 12. What gland is responsible for the colloquial or physiological condition called jet-lag secondary to lack of sufficient naturallyoccurring hormone (melatonin) secreted/produced by this gland? Pineal gland, pial capsule (correct answer, your response)

13. Crouzon Syndrome, also known as branchial arch syndrome, affects the first branchial (or pharyngeal) arch, which is the precursor of maxilla & mandible. Since the branchial arches are important developmental feature in a growing embryo, a disturbance in their development creates lasting & widespread effects either secondary to arterial supply to the branchial arch problem or denervation. In lieu with this syndrome, one of the cranial nerves below is considered as an epibranchial placodes contributing to ganglia for nerves of the pharyngeal arches. CN V (correct answer, your response) 14. Anlage of cerebellum is derived from combination of the metencephalon, alar plates & what important embryologic primitive brain structure? Rhombic lip (correct answer, your response) 15. All but one, are cranial nerves carrying parasympathetic visceral efferent to ganglia: Trigeminal nerve (correct answer, your response) 16. The corneal reflex is controlled by what Cranial Nerve/s? Trigeminal Nerve and Facial Nerve (correct answer, your response) 17. The following statements regarding the Vagus nerve is/are incorrect, EXCEPT: The vagus nerve motor nucleus innervates the constrictor muscle of the pharynx and the intrinsic muscles of the larynx (correct answer, your response) 18. Transection at the level of the Left midbrain will manifest with the following symptoms, EXCEPT: All of the above (correct answer, your response) 19. These cells main function is the insulation of the axons exclusively in the central nervous system of higher vertebrates, a function similarly performed by Schwann cells in the peripheral nervous system: Oligodendrocytes (correct answer, your response) 20. Another term for Pacchionis bodies arachnoid granulations, large & calcified (correct answer, your response) 21. The following statement/s concerning the trigeminal nerve is/are correct: The main motor nuclei of CN V is located in the pons (correct answer, your response) 22. Patient N.F., a 65 year-old known right-handed wealthy man was rushed to the E.R., dead-on-arrival. Three months & 2 weeks prior to incidence, patient was noted to have a very slow but progressive hemiparesis & hemiplegia of right extremities with progressive slurring of speech. Past Medical History reveals, (+) 33 pack-years of cigarette smoking & known Chronic Hypertensive with concomitant Diabetes Mellitus Type 2, Uncontrolled. Patients family (heir) from Finland requested for an autopsy of the brain to rule-out any foul-play. Autopsy

reveals: infarcted lateral surface & temporal pole of left cerebral hemisphere. Initial Impression: CVA, Middle Cerebral Artery, Left, Thrombotic (correct answer, your response) 23. All sensory impulses will be process and integrated in the thalamus, EXCEPT: Olfaction (correct answer, your response) 24. Red nucleus contains pigmented cells composed of what specific ion? iron (correct answer, your response) 25. This is a circular hole in the sphenoid bone of the skull base that connects the middle cranial fossa and the pterygopalatine fossa which permits passage of maxillary nerve branch of Trigeminal cranial nerve: Foramen Rotundum (correct answer, your response) 26. A 24 year old female, non diabetic, non hypertensive, consulted a neurologist because of facial deviation. History revealed that 1 week ago, she experienced a low grade fever accompanied by general body malaise. The next day, the patient noted asymmetry on facial movements deviating to the left, associated by difficulty in closing the right eyelid. After a thorough Physical examination and Neurological evaluation, she was diagnosed with Bells Palsy. The following regarding Bells Palsy is/are true except: The upper quadrant of the right side of the face is spared. (correct answer, your response) 27. Lateral expansions of the superior sagittal sinus of the dura mater, often increasing in width with advancing age until they extend at least 2 centimetres lateral to the midline. lateral venous lacunae (correct answer, your response) 28. Schizencephaly can be distinguished from Porencephaly by the fact that in schizencephaly, the distorted/deformed cortical part of the brain is lined with ___________ . gray matter (correct answer, your response) 29. Chief markers of Goldenhar Syndrome are incomplete development of ear, nose, soft palate, lip, and mandible on usually one side of the face, hence other term is hemifacial microsomia. Additionally, there are issues with internal organs underdevelopment and manifestations of scoliosis, lipodermoids (fat in the eye) & sensorineural hearing loss. Other term for Goldenhar Syndrome: Occulo-Auriculo-Vertebral Syndrome (correct answer, your response) 30. Dandy Walker Syndrome is a congenital brain malformation involving the cerebellum & the fluid filled spaces around it. The key feature of this syndrome is an enlargement of the 4th ventricle but with regards to 2nd type of Dandy Walker Syndrome, the fourth ventricle is not abnormally dilated. The 2nd type of DWS is represented by a large accumulation of CSF in the posterior fossa secondary to an enlarged

actual space, known as: Cisterna Magna (correct answer, your response) 31. Its primary division segregates into future diencephalon and secondary division, from which alar plates protrude to, generates the development of telencephalon: Procencephalon (correct answer, your response) 32. The cranial nerve/s involved in pupillary light reflex is/are: CN II and III (correct answer, your response) 33. Loeys-Dietz Syndrome is a recently-discovered autosomal dominant genetic syndrome which has many features caused by mutations in the genes encoding the transforming growth factor beta receptor 1 or 2 which results to clinical characteristics such as hypertelorism, cleft palate or bifid uvula & aortic & arterial aneurysms/dissections with turtousity (corkscrew structure) of the arteries. Define hypertelorism: None of these (correct answer, your response) 34. The following statements regarding the midbrain is correct, EXCEPT: The substantia nigra separates the Tegmentum and the Tectum of the midbrain (correct answer, your response) 35. The anterior pituitary which synthesizes & secretes important endocrine hormone, such as ACTH, TSH, PRL, GH, endorphins, FSH & LH, develops from a depression in the dorsal wall of the pharynx (stomodeal part) known as _____________ . Rathkes pouch (correct answer, your response) 36. The following statements concerning the colliculi of the midbrain is/are correct, EXCEPT: They are located in the Tegmentum (correct answer, your response) 37. All but one, are branches of Posterior Circulation, clinically: Posterior Communicating Artery (correct answer, your response) 38. ANALOGY: Arnold-Chiari Syndrome is to _______________; while, Budd-Chiari Syndrome is to ________________. Cerebellar tonsils; Hepatic vein (correct answer, your response) 39. According to normal cerebrospinal fluid flow & resorption, this is the only mesencephalic duct which contains cerebral fluid which connects the 3rd ventricle in the diencephalon to the 4th ventricle, which is between the pons & cerebellum. Name this communicating canal: Sylvius (correct answer, your response) 40. The following statements are true, EXCEPT: The fibers of Corticospinal tract at the level of the facial colliculus are crossed fibers (correct answer, your response) 41. These are valveless veins which normally drain external veins of the skull into the dural venous sinuses. However, because they are valveless, pus can flow into the skull through them as well, making them a possible route for transmission of extracranial infection to get into the skull.

emissary (correct answer, your response) 42. The nuclei associated with the facial nerve include the following, EXCEPT: Nucleus ambiguus (correct answer, your response) 43. The medial surface of the thalamus forms the superior part of the lateral wall of the third ventricle and is usually connected to the opposite thalamus by a band of gray matter known as: Both A and B (correct answer, your response) 44. The following statements are true, EXCEPT: The fibers of the medial lemniscus are uncrossed fibers (correct answer, your response) 45. The cranial nerves listed below are associated with the following functions, EXCEPT: The abducens nerve moves the ipsilateral eye medially (correct answer, your response) 46. A 65 year old female, non diabetic, non hypertensive, was noted to have a progressive loss of vision on both eyes. The symptom was associated with episodes of vomiting, alteration in body temperature, most of the time febrile. Blood chemistry, complete blood count, urinalysis and chest x-ray were unremarkable. CT scan of the cranium was done revealing a 5x5 cm circumscribed mass. With your knowledge in neuroanatomy, what is the most probable site of the lesion? Optic Chiasm (correct answer, your response) 47. What formed zone around the neuroepithelial layer is responsible for the formation of white matter? marginal layer (correct answer, your response) 48. The Transverse Sinuses become the Sigmoid Sinuses as they approach what specific structure/s: Petrous posterior aspects of temporal bones (correct answer, your response) 49. Another term for Cisterna Magna: posterior cerebellomedullary cistern (correct answer, your response) 50. If you have a high index suspicion of increased intracranial pressure on your patient, you should elicit first what very important objective physical finding? papilledema (correct answer, your response) 51. The following cranial nerve will move the eye,: CN III, IV and VI (correct answer, your response) 52. The second largest Dural reflection: Cerebellar Tentorium (correct answer, your response) 53. Samters Syndrome is a triad of illnesses suffered by a patient. Triad of Bronchial Asthma, Aspirin sensitivity/allergy, and nasal or ethmoidal polyposis. Polyposis under this syndrome starts from a symptom of allergic rhinitis with progressive anosmia. Define anosmia:

Lack of CN I function (correct answer, your response) 54. Blood received by the Confluence of Sinuses is drained by what dural venous sinus? Transverse Sinus, Left > Right (correct answer, your response) 55. This central nervous system gland is large in children, but shrinks at puberty. It appears to play a major role in sexual development, hibernation in animals, metabolism, and seasonal breeding. The abundant melatonin levels in children produced by this gland (also known as epiphysis), are believed to inhibit sexual development. Tumors to this organ have been linked with precocious puberty. When puberty arrives, melatonin production is reduced. Calcification of this gland is typical in adults located on the roof of the mesencephalon; hence, it then serves as a landmark on an x-ray of the skull: Pineal Body (correct answer, your response) 56. The following statements regarding the posterior surface of the pons is/are correct, EXCEPT: substantia nigra (correct answer, your response) 57. A 50 year old male, known Diabetic, known hypertensive, experienced sudden loss of sensation on Right side of the body. Motor strength examination revealed no signs of paresis or paralysis on both sides of the body. Other neurological examinations were unremarkable. What is the most probable site of the lesion? Left Thalamus (correct answer, your response) 58. Also known as the accessory oculomotor nucleus, which is the accessory parasympathetic cranial nerve nucleus of the oculomotor nerve, supplying the constricting muscles of the iris resulting to constriction/dilatation of both pupils & accommodating their lenses: Edinger-Westphal nucleus (correct answer, your response) 59. A case of 8 months old, male assisted by mother in your clinic with a chief complaint of well-defined ring of non-scarring alopecia (isolated hair loss) with normal head circumference upon physical examination. Personal & Social History reveals that parents shaved the childs head below the ring of thinned hair in an effort to make the hair loss less noticeable. A doctor before you already diagnosed the patient with Halo-Scalp Ring. Maternal History reveals that the patient was born to a 24-year-old primigravida (first-time pregnancy) who underwent 20 hours of extreme labor prior a successful vaginal delivery. You scheduled the patient for a follow-up after one week with babys Pedia neonatal history & P.E. record. First cephalic condition to look for in the history right after birth: caput succedaneum (correct answer, your response) 60. Great Cerebral Vein + Inferior Sagittal Vein = _______________. Straight Sinus (correct answer, your response) 61. Expalin why white matter is the white tissue of the freshly cut brain? Because of the fatty substance called myelin that surrounds nerves

axon. (correct answer, your response) 62. A 23 year old male medical student was involved in a vehicular accident a week ago. During his follow up consult to his attending physician, he complained of difficulty raising his left shoulder. On physical examination, all were unremarkable except for gross weakness in left shoulder elevators. With your knowledge in Anatomy, what would be your initial impression? Left Spinal accessory nerve must be damaged (correct answer, your response) 63. The following statements concerning the internal structures of the midbrain is/are correct, EXCEPT: The central gray matter encircles the red nuclei (correct answer, your response) 64. Another term for skullcap calvaria (correct answer, your response) 65. Every thalamic nuclei sends axons to different parts of the cerebral cortex, EXCEPT: Reticular Nucleus (correct answer, your response) 66. This is a cephalic disorder that results from a neural tube defect secondary to maternal folate deficiency that occurs when the cephalic (head) end of the neural tube fails to close, usually between the 23rd & 26th day of pregnancy, resulting in the absence of a major portion of the brain, skull & scalp: Anencephaly (correct answer, your response) 67. This is a rare neural tube defect that combines extreme retroflexion of the head with severe defects of the spine. And newborns with this defect seldom live more than a few hours: iniencephaly (correct answer, your response) 68. These plates in the brain form sensory tracts in myelencephalon & metencephalon including the olivary nuclei & solitary nuclei: alar plates (correct answer, your response) 69. The following statements concerning the pons is/are correct, EXCEPT: The whole length of the pons is traversed by the cerebral aqueduct posteriorly (correct answer, your response) 70. The following are the component structures of the epithalamus, EXCEPT: None of the above (correct answer, your response) X. NEUROANATOMY MODULE PART 2 1. The following statement/s is/are true regarding ascending tracts of the spinal cord except: Sensory information originating outside the body such as pain, temperature and touch are known as Proprioception (correct answer, your response) 2. Pathway subserving the position and vibration sense of the legs:

Fasciculus gracilis (correct answer, your response) 3. The fibers of the posterior columns are arranged in the following order from lateral to medial: Cervical segments, thoracic, lumbar & sacral segments (correct answer, your response) 4. Which of the following cerebellar peduncles exclusively carry afferent axons? middle (correct answer, your response) 5. What is the second order neuron of the fasciculus gracilis? Nucleus Gracilis (correct answer, your response) 6. The following statements concerning the Corticospinal tracts are correct except: Those that control the movements of the upper limb originate in the precentral gyrus on the medial side of the cerebral hemisphere (correct answer, your response) 7. The deep cerebellar nuclei receiving inputs from the cerebral cortex and participates in the planning of movement: dentate (correct answer, your response) 8. The following findings are consistent with cerebellar disease, EXCEPT: resting tremors (correct answer, your response) 9. Which of the following cells has an excitatory synaptic action? granule (correct answer, your response) 10. RC, a 26 year old male sustain a gunshot wound hitting his 10th thoracic vertebra. On physical examination, the following findings were noted: Paralysis of both lower extremity musculature Loss of sensation to needle prick, hot and cold, deep pressure on both lower extremities (-) vibratory sense on both lower extremities Reflexes (absent on both lower extremities) (-) bulbocavernosus reflex With your knowledge in neuroanatomy, what kind of cord transection is most likely? Complete cord syndrome (correct answer, your response) 11. The layer of the cerebral cortex predominantly made of dendrites is: Layer I (correct answer, your response) 12. A spinal cord syndrome that can present initially as a hemi paresis is called: Brown Sequard Syndrome (correct answer, your response) 13. The following are tracts found in the lateral white column: Anterior and posterior spinocerebellar (correct answer, your response) 14. Which of the following lobes of the cerebellum is important in the control of limb and trunk movements?

anterior (correct answer, your response) 15. The group of nerve cells of the anterior gray column that is present in most segments of the spinal cord and is responsible for innervating the skeletal muscles of the neck and trunk is the __________ group: Medial (correct answer, your response) 16. The predominant layer of the cerebral cortex of the post central gyrus is: Layer IV (correct answer, your response) 17. The following statements regarding the Corticospinal tracts are true except: None of the above (correct answer, your response) 18. Which of the following cerebellar cortical neurons axons form parallel fibers? granule (correct answer, your response) 19. The following is/are component/s of the reflex arc except: None of the above (correct answer, your response) 20. The Climbing fibers project to which of the following cerebellar cortical neurons? purkinje (correct answer, your response) 21. Which of the following lobes of the cerebellum is important in maintaining balance and controlling eye movement? flocculonodular (correct answer, your response) 22. The following statements regarding descending tracts are false except: The 3rd order neuron of the descending tracts is the thalamus (correct answer, your response) 23. The following pathways / structures are going to be injured in anterior spinal artery infarctions EXCEPT: Fasciculus Cuneatus and Gracilis (correct answer, your response) 24. The amount of gray matter is greatest in the following segments of the cord: Cervical and lumbosacral (correct answer, your response) 25. There are ______ pairs of spinal nerves attached along the entire length of the spinal cord: 31 (correct answer, your response) 26. Component of the cerebral hemisphere concerned with learning and memory: hippocampal formation (correct answer, your response) 27. LC, 56 year old male, smoker, diabetic, hypertensive, consulted a private physician because of progressive weakness of right upper extremity and right lower extremity. Despite the presence of weakness in the lower extremity, he can still ambulate without any difficulties. The symptom was also accompanied by numbness in the right upper extremity. No sensory deficit noted in the lower extremities. If you were the attending physician, where do you think the lesion is probably located?

Left lateral cerebral hemisphere (correct answer, your response) 28. Based on the sensory homunculus, which of the following is supplied by the anterior cerebral artery? genitals (correct answer, your response) 29. The following deep cerebellar nuclei are collectively termed interposed nuclei: globose & emboliform (correct answer, your response) 30. VS, 49 year old female hypertensive non diabetic was brought to the emergency room because of sudden paralysis of right side of the body. Physical examination revealed the following: Weakness of Right upper extremity and right lower extremity 100% loss of sensation on all aspects of sensory modalities In the case presented above, what neuronal order of the Corticospinal tract was severely injured in the patient? First order neuron 31. Spinal cord pathway subserving sensation for light touch in the arms: Ventral Spinothalamic tract (correct answer, your response) 32. The main blood supply of the spinal cord comes from the ____________ artery: Anterior spinal 33. The structure that arises from the end of the spinal cord as it tapers off and descends to be attached to the posterior surface of the coccyx is called: Filum terminale (correct answer, your response) 34. The Climbing fibers of the cerebellar cortex originate from which of the following nuclei? inferior olivary complex (correct answer, your response) 35. The following best describes the posterior cord syndrome of the spinal cord: Bilateral Proprioceptive sensory loss below the level of the lesion 36. Based on the motor homunculus, the lower extremities are supplied by which of the following cerebral arteries? anterior cerebral Y. NEUROANATAMOY MODULE PART 3

Você também pode gostar